You are on page 1of 90

Copyright © 2014 Delhi Academy of Medical Sciences, All Rights Reserved.

1/90
Test Information
Test Name SWTS-DERMATOLOGY & PSYCHIATRY 2017 (MDMS) Total Questions 200

Test Type Examination Difficulty Level Difficult

Total Marks 600 Duration 120minutes

Test Question Language:- ENGLISH

(1). Hyper extensibility with normal elastic recoil is seen in

a. Ehlers-Danlos syndrome

b. Pseudoxanthoma elasticum

c. Cutis laxa

d. Scleroderma

Solution. (a) Ehlers-Danlos syndrome


Ref:Read the text below
Sol:
- Collagen defects allow the hyperextensibility of skin and loose jointedness but skin bounces back because of normal elastin as in Ehlers
Danlos syndrome.
- However, in elastin defects like cutis laxa and pseudoxanthoma elasticum skin looses its elastic recoil and hangs in loose folds.
- Scleroderma on the other hand is characterized by excessive abnormal collagen producing binding down of skin that is difficult to pinch
and pick into folds.

Correct Answer. a

(2). A 5 yr old male child has multiple hyperpigmented macules over the trunk. On rubbing the lesion with rounded end of pen, he developed
urticarial wheal, confined to border of lesion, most likely diagnosis is-

a. Fixed drug eruption

b. Lichen planus

c. Urticaria pigmentosa

d. Urticarial vasculitis

Solution. (c) Urticaria pigmentosa


Ref:Read the text below
Sol:
- It is urticaria pigmentosa.
- It is cutaneous mastocytosis presenting with pigmented lesions.
- On rubbing the lesions mast cells degranulate releasing histamine.
- This produces the wheal and flare of the lesion. It is known as Darier’s sign.

Correct Answer. c

Copyright © 2014 Delhi Academy of Medical Sciences, All Rights Reserved. 2/90
(3). Which of the following is a suprabasal split

a. Pemphigus vegetans

b. Pemphigus foliaceus

c. Fogo selvagem

d. Staphylococcal scalded skin syndrome

Solution. (a) Pemphigus vegetans


Ref:Read the text below
Sol:
- Pemphigus vulgaris and vegetans have suprabasal split.
- Pemphigus foliaceus and its variants like fogo selvagem, pemphigus erythematosus have subcorneal split.
- SSSS and bullous impetigo are mediated by epidermolytic toxins that cleave the desmoglein 1 and produce a subcorneal split.

Correct Answer. a

(4). Look at the clinical picture in Fig 1,What is the diagnosis.

a. Molluscum contagiosum

b. Warts

c. Cryptococcosis

d. Lichen planus

Solution. (a) Molluscum contagiosum


Ref: Read the text below
Sol :
-Molluscum contagiosum (MC) is a viral infection of the skin or occasionally of the mucous membranes, sometimes called water warts.
-It is caused by a DNApoxvirus called the molluscum contagiosum virus (MCV). MCV has no nonhuman-animal reservoir (infecting only
humans). -There are four types of MCV, MCV-1 to -4; MCV-1 is the most prevalent and MCV-2 is seen usually in adults. The virus that
causes molluscum is spread from person to person by touching the affected skin.

Correct Answer. a

Copyright © 2014 Delhi Academy of Medical Sciences, All Rights Reserved. 3/90
(5). Lines of Blaschko represent

a. Lines of embryonal migration

b. Lines of vessels

c. Lines of nerves

d. Lines of lymphatics

Solution. (a) Lines of embryonal migration


Ref: Read the text below
Sol :
-Lines of Blaschko represent the lines of migration of embryonal cells. Many mosaic skin disorders like nevi follow these lines.

Correct Answer. a

(6). Desmosomes connect

a. Keratinocyte and melanocyte

b. Keratinocyte and Langerhans cell

c. Keratinocyte and Merkel cell

d. Langerhans cell with melanocyte

Solution. (c) Keratinocyte and Merkel cell


Ref: Read the text below
Sol :
- Desmosomes connect two keratinocytes together.
- Desmosomal connections have also been identified between keratinocytes and Merkel cells.
- As a result Merkel cells are now believed to be ectodermal in origin despite the presence of neurosecretory granules arguing for a
neural crest origin.
- Langerhans cells and Melanocytes are intruders into the epidermis and are not connected with surrounding keratinocytes with
desmosomes.

Correct Answer. c

(7). Which of the following is not a feature of poikiloderma

a. Atrophy

b. Reticulate pigmentation

c. Telangiectasia

d. Desquamation

Solution. (d) Desquamation


Ref: Read the text below
Sol :
- Poikiloderma is a triad of atrophy, reticulate pigmentation and telangiectasia.
- Desquamation is scaling.
- Poikiloderma is typically seen in photosensitive disorders.

Correct Answer. d

Copyright © 2014 Delhi Academy of Medical Sciences, All Rights Reserved. 4/90
(8). The term of schizophrenia is coined by

a. Eugen Bleuler

b. Sigmund Freud

c. Franz Alexander

d. John weyer

Solution. (a) Eugen Bleuler


Ref: Read the text below
Sol :

Correct Answer. a

Copyright © 2014 Delhi Academy of Medical Sciences, All Rights Reserved. 5/90
(9). The patient shown in the Fig 2 presented with chief complaints of itching over left foot. This is an example of:

a. Organic hallucination

b. Functional hallucination

c. Illusion

d. Pseudo hallucination

Solution. (a) Organic hallucination


Ref: Read the text below
Sol:
- The phantom limb is the most common organic somatic hallucination of psychiatric origin. In this case the patient feels that they have a
limb from which in fact they are not receiving any sensations either because it has been amputated or because the sensory pathways
from it have been destroyed.
- In rare cases with thalamo−parietal lesions the patient describes a third limb. In most phantom limbs the phenomenon is produced by
peripheral and central disorders.
- Phantom limb occurs in about 95% of all amputations after the age of 6 years. Occasionally a phantom limb develops after a lesion of
the peripheral nerve or the medulla or spinal cord.
- The phantom limb does not necessarily correspond to the previous image of the limb in that it may be shorter or consist only of the
distal portion so that the phantom hand arises from the shoulder.
- If there is clouding of consciousness, the patient may be deluded that the limb is real.
- Equivalent perceptions of phantom organs may also occur after other surgical procedures such as mastectomy, enuleation of the eye,
removal of the larynx or the construction of a colostomy.
- The person is aware of the existence of the organ or limb and describes pain or paraesthesia in the space occupied by the phantom
organ and this persists in a minority of patients.
- When the experience is related to a limb the perception shrinks over time, with distal parts disappearing more quickly than those that
are proximal. Lesions of the parietal lobe can also produce somatic hallucinations with distortion or splitting-off of body parts.

Correct Answer. a

(10). Increase in goal directed activity is seen in

a. Grief

b. Mania

c. Depression

d. Schizophrenia

Solution. (b) Mania


Ref: Read the text below
Sol :
- A Manic Episode is defined by a distinct period during which there is an abnormally and persistently elevated, expansive, or irritable
mood. This period of abnormal mood must last at least 1 week
- The mood disturbance must be accompanied by at least three additional symptoms from a list that includes inflated self-esteem or
grandiosity, decreased need for sleep, pressure of speech, flight of ideas, distractibility, increased involvement in goal-directed activities
or psychomotor agitation, and excessive involvement in pleasurable activities with a high potential for painful consequences.

Correct Answer. b

Copyright © 2014 Delhi Academy of Medical Sciences, All Rights Reserved. 6/90
(11). Catatonic schizophrenia is associated with

a. Paranoid thoughts

b. Suicidal behavior

c. Motor symptoms

d. Mental Retardation

Solution. (c) Motor symptoms


Ref: Read the text below
Sol :
- Catatonic schizophrenia is a type (or subtype) of schizophrenia that includes extremes of behavior.
- At one end of the extreme the patient cannot speak, move or respond - there is a dramatic reduction in activity where virtually all
movement stops, as in a catatonic stupor.
- At the other end of the extreme they are overexcited or hyperactive, sometimes mimicking sounds (echolalia) or movements
(echopraxia) around them - often referred to as catatonic excitement.

Correct Answer. c

(12). The essential feature for the diagnosis of depression is

a. Dissociation

b. Motor symptoms

c. Anhedonia

d. Restlessness

Solution. (c) Anhedonia


Ref: Read the text below
Sol :
Individuals with persistent depressive disorder describe their mood as sad or “down in the dumps.” During periods of depressed mood, at
least two of the following six symptoms from are present.
- Poor appetite or overeating
- Insomnia or hypersomnia
-Low energy or fatigue
- Low self-esteem
- Poor concentration or difficulty making decisions
-Feelings of hopelessness

Correct Answer. c

(13). A45-year-old patient tells her doctor that after hearing that her husband died, she could not remember leaving her office and going
home.In every other respect, her memory is intact. Which of the following types of amnesia is this an example of?

a. Continuous amnesia

b. Retrograde amnesia

c. Localized amnesia

d. Generalized amnesia

Solution. (c) Localized amnesia


Reference: Read the text below
Sol:
- The choices in this question describe different patterns of amnesia. Localized amnesia refers to memory loss surrounding a discrete
period of time, typically occurring after a traumatic event; in this case, the patient’s amnesia results from learning of her husband’s
death.
- Selective amnesia involves the inability to recall certain aspects of an event, though other memories of the event may be intact.
- Rarely, a patient may forget his or her entire preceding life (generalized amnesia) or forget all events following a trauma, except for the
immediate past (continuous amnesia).
- Retrograde amnesia is any amnesia for events that come before a traumatic event.

Correct Answer. c

Copyright © 2014 Delhi Academy of Medical Sciences, All Rights Reserved. 7/90
(14). A commonly administered screening test to evaluate for dementia is which of the following?

a. Geriatric Rating Scale

b. Glasgow Coma Scale

c. Folstein Mini-Mental Status Examination (MMSE)

d. Mental Status Examination (MSE)

Solution. (c) Folstein Mini-Mental Status Examination (MMSE)


Reference: Read the text below
Sol:
- The Folstein MMSE is a frequently used screening assessment for dementia. It is a 30-point scale with deductions for incorrect answers.
- The Glasgow Coma Scale is an easy-toperform instrument that evaluates level of consciousness.There are three general categories that
the examiner tests: eye opening, verbal response, and best motor response.
- Each category receives a number for patient response.Overall scores range from 3 to 15, with lower scores reflecting more severely
impaired consciousness.The Geriatric Rating Scale is a rating scale for nonprofessional staff to evaluate patients’ abilities to perform
their activities of daily living and interact with others.
 It may be most helpful in evaluation of the moderately to severely demented individual. The Blessed Rating Scale is a tool that typically
asks a patient’s friends or relatives to assess the ability of the patient to function in his or her current environment.
 An MSE is the formal psychiatric examination that includes appearance, assessment of mood and affect, presence of psychosis,and
evaluation of insight and judgment. It is not a substitute for the Folstein MMSE.

Correct Answer. c

(15). Pseudo isomorphic response is seen in

a. Vitiligo

b. Psoriasis

c. Lichen planus

d. Warts

Solution. (d) Warts


Ref: Read the text below
Sol :
- True isomorphism (Koebner’s) is seen in vitiligo, psoriasis and lichen planus while pseudo isomorphism is seen in autoinoculation in
warts and molluscum contagiosum.

Correct Answer. d

(16). Choose the correct sentence

a. Crystalline miliaria is due to obstruction of most superficial part of eccrine sweat duct

b. Fordyce disease is associated with apocrine glands

c. Palms and soles have eccrine glands solely controlled by thermal stimuli

d. Sebaceous glands are mostly free but sometimes may be associated with hair follicles

Solution. (a) Crystalline miliaria is due to obstruction of most superficial part of eccrine sweat duct
Ref: Read the text below
Sol :
- Blockage of eccrine glands produces miliaria.
- Most superficial type produces crystalline miliaria while mid level block produces miliaria rubra (prickly heat) and deep blocks produce
miliaria profunda.
- Sebaceous glands are always associated with hair follicles except free/ectopic glands like Fordyce disease or granules.
- Eccrine sweat glands of palms and soles differ from other glands by responding to emotional stimuli too.

Correct Answer. a

Copyright © 2014 Delhi Academy of Medical Sciences, All Rights Reserved. 8/90
(17). Fish handler’s disease is

a. Swimming pool granuloma

b. Fish tank granuloma

c. Erysipeloid

d. Sporotrichosis

Solution. (c) Erysipeloid


Ref: Read the text below
Sol :
- Fish handler’s disease is erysipeloid caused by Erysipelothrix spp.
- Fish tank granuloma and swimming pool granuloma are cutaneous atypical mycobacteriosis caused by M. marinum.
- Sporotrichosis is known as rose gardener’s disease.

Correct Answer. c

(18). Defect in gene ATP2C1 is associated with

a. Hailey-Hailey disease

b. Darier’s disease

c. Grover’s disease

d. Dermatitis herpetiformis

Solution. (a) Hailey-Hailey disease


Ref:Read the text below
Sol:
Hailey-Hailey disease
- Autosomal dominant inheritance
- Defect in gene ATP2C1, (ATPase, Ca2+ transporting, type2C, member1,
a regulator of cytoplasmic calcium concentration) leads to acantholysis.
- Clinically presents in 2nd to 3rd decade with recurrent intertrigo with
vesicles, erosions, crusts, erythema, maceration and foul smell in intertriginous areas like groin, axillae and neck.
- It can be confused with intertrigo, candidiasis & tinea cruris
Histology: Extensive epidermal acantholysis in mid epidermis with “dilapidated or crumbling brick wall” appearance. Darier’s disease is
associated with ATP2A2 defect

Correct Answer. a

(19). A female sex worker presents with vaginal discharge. In absence of speculum examination which conditions are to be treated for in
syndromic management:

a. Gonorrhea + Syphilis

b. Gonorrhea + Syphilis + Candidiasis

c. Gonorrhea + Syphilis + Candidiasis + Trichomonas vaginalis

d. Gonorrhea + Syphilis + Candidiasis + Trichomonas vaginalis + Bacterial Vaginosis

Solution. (d) Gonorrhea + Syphilis + Candidiasis + Trichomonas vaginalis + Bacterial Vaginosis


Ref:Read the text below
Sol:
- In Syndromic approach of vaginal discharge if patient or partner is high risk for STDs or if speculum examination can not be done then
both vaginal discharge (Trichomonas, candida, bacterial vaginosis) and cervical discharge (gonorrhea and chlamydia).

Correct Answer. d

Copyright © 2014 Delhi Academy of Medical Sciences, All Rights Reserved. 9/90
(20). School of fish appearance is seen in

a. Chancroid

b. Hunterian chancre

c. Donovanosis

d. Mycobacterial Granuloma

Solution. (a) Chancroid


Ref:Read the text below
Sol:
- Chancroid is an acute STD characterized by a painful ulcer at the site of inoculation, usually on the external genitalia, and the
development of suppurative regional lymphadenopathy.
- Gram’s Stain Of scrapings from ulcer base or pus from bubo, show small clusters or parallel chains of gram-negative rods (School of
Fish appearance). Interpretation is difficult due to presence of contaminating organisms in ulcers.
- Hunterian chancre is syphilitic.
- Donovanosis (Granuloma inguinale) is a mildly contagious, chronic, indolent, progressive, autoinoculable, ulcerative disease involving
the skin and lymphatics of the genital and perianal areas.
- It is caused by Calymmatobacterium granulomatis, an encapsulated intracellular gram-negative rod. Touch or Crush Preparation
- Of punch biopsy stained withWright’s or Giemsa’s stain shows Donovan bodies in cytoplasm of macrophages. Mycobacterial infections
do not have any school of fish appearance.

Correct Answer. a

Copyright © 2014 Delhi Academy of Medical Sciences, All Rights Reserved. 10/90
(21). Which Vitamin deficiency can produce the typical skin change shown in Fig 3 ?

a. Vitamin B1

b. Vitamin B2

c. Vitamin B3

d. Vitamin B6

Solution. (c) Vitamin B3


Ref:Read the text below
Sol:
- Casal’s necklace pattern of skin lesion around the neck is a feature of Pellagra.
- The recommended daily allowance of Niacin is 6mg/1000 kcal of energy intake.

Correct Answer. c

(22). In a 29-year-old man being treated for depression,which of the following statements would most likely characterize his sleep patterns?

a. His sleep patterns are unaffected.

b. He has few if any episodes of early morning awakening.

c. He experiences less REM latency.

d. His REM sleep tends to be distributed to the latter half of the night.

Solution. (c) He experiences less REM latency.


Reference: Read the text below
Sol:
- Abnormal sleep patterns are common in depression and may be the chief complaint of the depressed patient.
- Classically, increased sleep latency (difficulty falling asleep), early morning awakening, and increased wakefulness are seen.
- REM sleep tends to be redistributed so that most of it occurs in the first half of the night.
- REM latency (the period of time between falling asleep and starting REM sleep) decreases.

Correct Answer. c

Copyright © 2014 Delhi Academy of Medical Sciences, All Rights Reserved. 11/90
(23). A26-year-old man with narcolepsy explains that he has episodes of brief paralysis without any loss of consciousness or other deficits.
These attacks are usually precipitated by laughter or anger. What is this phenomenon known as?

a. Cataplexy

b. Hypnagogic hallucinations

c. Hypnopompic hallucinations

d. Catalepsy

Solution. (a) Cataplexy


Reference: Read the text below
Sol:
- Narcolepsy is a sleep disorder characterized by the tetrad of hypersomnia (excessive daytime somnolence), cataplexy (transient loss of
motor tone associated with strong emotions), sleep paralysis (total or partial paralysis in sleep-wake transition), and hypnagogic
hallucinations (vivid dream-like hallucinations occurring in the wake-sleep transition).
- Hypnopompic hallucinations occur in the sleep-wake transition.
- Catalepsy is a state of immobility sometimes seen in catatonic states.

Correct Answer. a

(24). A 22-year-old man is brought to the emergency department 2 weeks after a motor vehicle accident in which he suffered significant head
trauma. His parents are concerned because, although the patient recognizes them on the telephone and responds appropriately, when he
sees them face-to-face, he believes them to be imposters who have replaced his real parents. He agrees that these people look like his
real parents but is convinced that they are indeed imposters. What delusion is this known as?

a. Cacodemonomania

b. Doppelganger

c. Capgras syndrome

d. Delusion of reference

Solution. (c) Capgras syndrome


Reference: Read the text below
Sol:
- Capgras syndrome is the delusional idea that imposters have replaced once familiar persons. This belief is held despite the
acknowledgment that the people appear exactly the same.
- Doppelganger is the belief in the existence of an identical counterpart.
- Cacodemonomania is the delusion of being poisoned by an evil spirit.

Correct Answer. c

(25). All are true about night terrors except

a. Parasomnia

b. Common in preschool children

c. Family history common

d. During REM sleep

Solution. (d) During REM sleep


Ref: Read the text below
Sol :
- A night terror, sleep terror or pavor nocturnus is a parasomnia disorder, causing feelings of terror or dread, and typically occurring in
the first few hours of sleep during stage 3 or 4 non-rapid eye movement (NREM) sleep.
-Night terrors tend to happen during periods of arousal from delta sleep, also known as slow-wave sleep.
- During the first half of a sleep cycle, delta sleep occurs most often, which indicates that people with more delta sleep activity are more
prone to night terrors. However, they can also occur during daytime naps

Correct Answer. d

Copyright © 2014 Delhi Academy of Medical Sciences, All Rights Reserved. 12/90
(26). Study the Fig 4 and comment on the diagnosis?

a. Sturger Weber syndrome

b. Tuberous sclerosis

c. Neurofibromatosis-1

d. Von Hippel Lindau syndrome

Solution. (b) Tuberous sclerosis


Ref:Read the text below
Sol:
- The images show adenoma sebaceum and ash leaf macule seen in Tuberous sclerosis.
- Most reliable early cutaneous sign of Tuberous sclerosis is Ash leaf macule
- Most commonly recognised cutaneous marker of Tuberous sclerosis is Adenoma sebaceum.

Correct Answer. b

(27). Clofazimine therapy can cause all except

a. Skin pigmentation

b. Icthyosis

c. Subacute Intestinal obstruction

d. Peripheral neuropathy

Solution. (d) Peripheral neuropathy


Ref: Read the text below
Sol:
- Peripheral neuropathy (Motor) is a complication of dapsone while important adverse reactions of clofazimine include a benign, reddish
cutaneous pigmentation which is reversible, Xerosis (Icthyosis), GI disturbances with severe diarrhea at high doses due to deposition of
dye crystals in mucosa and mesentery which may go to Intestinal obstruction.

Correct Answer. d

Copyright © 2014 Delhi Academy of Medical Sciences, All Rights Reserved. 13/90
(28). A 28 year old male presents with greasy, warty, hyperkeratotic papules over central chest and upper back. There are also nicks at the
free margin of some of the nail plates. Skin biopsy reveals acantholysis with presence of dyskeratotic cells in stratum corneum as well as
in stratum spinosum. The diagnosis is:

a. Truncal acne vulgaris

b. Seborrheic dermatitis

c. Dyskeratosis congenita

d. Darier’s disease

Solution. (d) Darier’s disease


Ref: Read the text below
Sol:
Darier’s disease is a rare autosomal dominant disease most likely caused by defects in ATPase2A2 calcium channel pump. It is
characterized by
1. Acantholysis (primary)
2. Pruritic, hyperkeratotic, warty, malodorous papules and plaques with greasy brownish scales in Seborrheic distribution
3. Corps ronds (in stratum spinosum) and grains (stratum corneum) on histology which are dyskeratotic keratinocytes.
4. Nail changes including longitudinal red and white streaks, V-shaped distal notching, and subungual hyperkeratosis.

Correct Answer. d

(29). A patient of Parkinsonism presents with erythema and scaling over widespread areas of body. The scales are likely to be

a. Greasy, yellow and semi-adherent

b. Adherent with follicular plugs

c. Silvery white and Auspitz sign positive

d. Furfuraceous and bran like

Solution. (a) Greasy, yellow and semi-adherent


Ref: Read the text below
Sol:
- Seborreheic dermatitis has such scales and the incidence of seborrheic dermatitis is increased in HIV, Parkinsonism, Downs’ syndrome
and other cerebrovascular events.

Correct Answer. a

(30). Topical calcineurin inhibitors are approved for the management of?

a. Atopic dermatitis

b. Seborrheic dermatitis

c. Nummular dermatitis

d. Irritant contact dermatitis

Solution. (a) Atopic dermatitis


Ref: Read the text below
Sol:
- Topical calcineurin inhibitors (Tacrolimus, Pimecrolimus) are approved for the management of atopic dermatitis especially in the
maintenance phase.
- However, increasingly they are being used for other therapeutic indications as

Correct Answer. a

Copyright © 2014 Delhi Academy of Medical Sciences, All Rights Reserved. 14/90
(31). Which of the following diseases is not due to antibodies against BP Ag 1 or 2

a. Bullous pemphigoid

b. Linear Ig A disease

c. Cicatricial pemphigoid

d. Epidermolysis bullosa acquisita

Solution. (d) Epidermolysis bullosa acquisita


Ref: Read the text below
Sol:
- Epidermolysis bullosa acquisita is due to IgG antibodies against the type 7 collagen (anchoring fibrils which are deep in lamina
fibroreticularis.
- The involvement here is seen in salt split skin as immunofluroscence in the base.Other diseases like Bullous pemphigoid, Cicatricial
pemphigoid, Herpes gestetionis, Linear IgA disease and dermatitis herpetiformis have some involvement of Bullous pepmhigoid antigen 1
&2 ( BP Ag 1,2) which are a part of hemidesmosome in the upper part of lamina lucida.
- They show imuunofluroscence predominantly in the roof of the blister cavity.

Correct Answer. d

(32). Which direct immunofluroscence finding is incorrectly matched

a. Pemphigus vulgaris: Fish-net pattern of IgG in intercellular spaces

b. Bullous pephigoid: Linear IgG and C3 at dermoepidermal junction

c. Chronic bullous disease of childhood: Linear IgA at dermoepidermal junction

d. Dermatitis herpetiformis: Granular IgG deposits at the dermal papillary tips

Solution. (d) Dermatitis herpetiformis: Granular IgG deposits at the dermal papillary tips
Ref: Read the text below
Sol:
IgA deposits not IgG. DIF Findings in various VB dermatoses:
a. BP- Linear C3, IgG at DEJ
b. CP- Linear C3, IgG and A at DEJ
c. DH- Granular IgA, C3 at DEJ more at tips of dermal papillae
d. EBA-Linear IgG, A, etc at DEJ
e. LAD and HG- Linear IgA C3 at DEJ
f. PF- IgG, C3 in the intercellular spaces of epidermis at granular layer
g. PV- IgG, C3 in the intercellular spaces of epidermis in spinous layer (FISH-NET pattern)

Correct Answer. d

(33). Hailey-Hailey disease is characterized by all except

a. Intertriginous area involvement

b. Dilapidated brick wall appearance on histopathology

c. Row of tombstone appearance on histology

d. Abnormalities of ATPase2C1

Solution. (c) Row of tombstone appearance on histology


Ref: Read the text below
Sol:
- Row of tombstone appearance is seen on histology of Pemphigus vulgaris.Hailey-Hailey disease presents as recurrent intertrigo in
multiple family members.
- It is due to defective calcium pump as there is mutation of ATPase2C1.
- It leads to acantholysis and a “dilapidated bick wall” appearance on histology.
- It is also kown as benign familial pemphigus.

Correct Answer. c

Copyright © 2014 Delhi Academy of Medical Sciences, All Rights Reserved. 15/90
(34). Secondary syphilis lesions are characterized by all except

a. Symmetrical eruption

b. Palm/ sole involvement

c. Severe pruritus

d. Rebound tenderness

Solution. (c) Severe pruritus


Ref: Read the text below
Sol:
- Lesions of secondary syphilis are usually asymptomatic except the rare follicular syphilid which may be pruritic.
- The lesions are polymormhous,symmetrical, have rebound tenderness (Bushke Olendroof or BO sign) and involve the palms and soles.

Correct Answer. c

(35). Flaccid pemphigus like blisters can be caused by all except

a. Captopril

b. Rifampicin

c. Penicillin

d. Furosemide

Solution. (d) Furosemide.


Ref: Read the text below
Sol:
- Flaccid pemphigus like blisters are produced by Captopril, Rifampicin, ACE inhibitors, Penicillins, Penicillamine and Piroxicam.
- Furosemide and 5-Flurouracil produces tense bullous pemphigoid like blisters.

Correct Answer. d

(36). Which is not due to a decrease in number of melanocytes

a. Vitiligo

b. Piebaldisim

c. Waardenberg syndrome

d. Albinism

Solution. (d) Albinism.


Ref: Read the text below
Sol:
- Vitiligo, piebaldism and waardenburg syndrome are due to loss of melanocytes.
- Albinism is due to decreased melanin production by melanocytes but they are not reduced in number.

Correct Answer. d

Copyright © 2014 Delhi Academy of Medical Sciences, All Rights Reserved. 16/90
(37). A 50 year old man complaints of painful nodule on the upper surface of ear shown in Fig 5 which is very tender.What is the most
probable diagnosis?

a. Keratoacanthoma

b. Pseudocyst of auricle

c. Chondrodermatitis nodularis helicis

d. Basal cell carcinoma

Solution. (c) Chondrodermatitis nodularis helicis


Ref:Read the text below
Sol:
- In Chondrodermatitisnodularishelicis,small painful nodules appear in the free border of helix in men about the age of 50 years.

Correct Answer. c

(38). Sweat glands are innervated by :-

a. Sympathetic fibers with Acetylcholine as neurotransmitter

b. Parasympathetic fibers with Acetylcholine as neurotransmitter

c. Sympathetic fibers with Adrenaline as neurotransmitter

d. Both Sympathetic & Parasympathetic fibers with Acetylcholine as neurotransmitter.

Solution. (a) Sympathetic fibers with Acetylcholine as neurotransmitter


Ref: Read the text below
Sol:
- This is very peculiar about sweat glands that though they are having sympathetic innervation but the neurotransmitter is Acetylcholine.
- Also remember that all postganglionic parasympathetic & few postganglionic.
- Sympathetic (sweat glands & some blood vessels) are having Muscarinic receptors while ganglia of both sympathetic &
parasympathetic are having nicotinic receptors

Correct Answer. a

Copyright © 2014 Delhi Academy of Medical Sciences, All Rights Reserved. 17/90
(39). Child with erythematous non blanching bosselated lesion on right side of face, treatment is

a. Erbium laser

b. Nd-YAG laser

c. Flash light pumped dye laser

d. Q ruby laser

Solution. (c) Flash light pumped dye laser


Ref: Dermatology in a Week. Premanshu Bhushan. Jaypee brothers. Pages: 148, 156 .
Sol:
The non blanching lesion with irregular bosselated surface is most likely a NEVUS FLAMMEUS (PORT-WINE STAIN =PWS) :
hemangiomas blanch (emptying sign)
- Capillary vascular malformation (not true neoplasm) consisting of increased numbers of dilated dermal vessels.
- Present at birth
- No spontaneous resolution (NB. Axial lesions tend to regress eg Stork bite; see below.)[PWS Persists]
- Macular in infancy-grow with patient-develop nodular/ papular areas (bosselated) and become darker in color.
- Treatment: Q switched Flash lamp pumped Pulsed Dye Laser (585 nm) treatment is most preferred, cosmetic camouflage. Lesions on
face require ophthalmologic consultation to look for Sturge Weber syndrome.
Variants of PWS:
- Stork bite / Salmon patch: occur in 30% of infants over occipital nuchal region & eyelids/glabella and usually (but not always) fade
within 1-2 years.
- Sturge-Weber syndrome: see above in genodermatoses.
- Klippel-Trenaunay-Weber syndrome: PWS lesions on limb with soft tissue and bony hypertrophy.
Main uses of Other lasers:
- Erbium laser/ CO2 laser: Resurfacing and ablation of raised lesions
- Nd: YAG laser: To reduce pigmentation (532 nm = Frequency doubled Nd:YAG or KTP laser for epidermal pigmentation like Freckles
and 1064 nm for deeper pigmentation like Nevus of Ota, Tatto etc mainly in Q-switched mode.
- Long pulsed Nd:YAG: Hair removal
- Flash lamp pumped Pulsed Dye laser: for vascular lesions like PWS, hemangiomas
- Ruby laser: For pigmentary lesions eg tattoo, hair removal
- Argon laser: Vascular lesions
- Excimer lasers: Vitiligo, Psoriasis

Correct Answer. c

Copyright © 2014 Delhi Academy of Medical Sciences, All Rights Reserved. 18/90
(40). Fine reticular pigmentation with palmar pits are seen in

a. Dowling–Degos disease

b. Rothmund Thompson syndrome

c. Cockyane syndrome

d. Bloom’s syndrome

Solution. (a) Dowling–Degos disease


Ref: Read the text below
Sol:
The correct answer is Kitamura’s reticulate acropigmentation but that is not one of the choices. The next best answer is dowling degos
disease. The relevant details are:
- DOWLING–DEGOS DISEASE : This rare autosomal dominant condition caused by loss of function mutations in the keratin 5 gene
(KRT5) usually presents postpubertally with asymptomatic, symmetrical, progressive, reticulate pigmentation of the flexures like axillae
and groins. Nails and hair are normal. Other features that may be present include scattered comedo-like lesions (dark dot follicles) and
pitted acneiform scars near the angles of the mouth. The histology is diagnostic, with a distinctive form of acanthosis characterized by an
irregular elongation of thin branching rete ridges, with a concentration of melanin at the tips with melanocyte count being normal.
- RETICULATE ACROPIGMENTATION OF KITAMURA: Reticulate acropigmentation of Kitamura is characterized by a reticulate
pigmentation on the dorsa of the hands, palmar pits and breakages of epidermal ridge pattern are found. Overlap of Kitamura and
Dowling degos is reported.
- SYMMETRICAL DYSCHROMATOSIS OF THE EXTREMITIES: RETICULATE ACROPIGMENTATION OF DOHI: Symmetrical
dyschromatosis of the extremities is an autosomal dominant disorder with mottled pigmentation and depigmentation on the dorsa of the
hands and feet and sometimes on the arms and legs.
- BLOOM SYNDROME is transmitted as an autosomal-recessive trait in Jews, characterized by photosensitive telangectatic erythema in
the butterfly area of the face and dwarfism in the first 2 years of life. Other changes that may be noted are bullous crusted lesions of lips,
cafe-au-lait spots, ichthyosis, acanthosis mgricans, syndactyly, irregular dentition, prominent ears, hypospadias, and cryptorchrdism. The
stunted growth is characterized by normal body proportions, no endocrine abnolmalities (except diabetes mellitus), and low birth weight
at full term. Dolichocephaly and narrow, delicate facies are present. Immune functions are abnormal, and gastrointestrnal and
respiratory infections occur frequently. Cancer of all cell types and sites are increased in frequency. Leukemia, lymphoma,
adenocarcinoma of the slgmoid colon, and oral and esophageal squamous cell carcinoma, as well as other malrgnancies, have been
associated with Bloom syndrome. About one-quarter of patlents under the age of 20 develop neoplasms. Photoprotection and sunscrrens
are recommended.
- ROTHMUND-THOMSON SYNDROME (POIKILODERMA CONGENITALE): Rothmund-Thomson syndrome is a rare autosomal-recessive
disorder. Poikiloderma begins at 3 to 6 months of age, with tense, pink, edematous patches on the cheeks and other exposed sites (acute
phase) followed by atrophy, reticulate pigmentation and telangectasia (chronic phase) typically involving the arms and legs are affected
with sparing of the antecubital and popliteal fossae. Other features: Short stature, saddle nose, absence or sparseness of eyebrows and
eyelashes, alopecia of the scalp, and numerous bone defects, hypogonadism, dystrophic nails, and defective dentition, Cataracts ,
occasional squamous and basal cell carelnoma of the skin , risk for osteosarcoma of bone is particularly high (>30%). At least a subset of
patients have abnormal DNA helicase activity.
- COCKAYNE SYNDROME: Autosomal recessive condition with photosensitivity and neurological degeneration. It differs from Xeroderma
pigmentosa by the lack of freckling and skin cancers and presence of dwarfism, beaked nose, and loss of subcutaneous fat, basal
ganglion calcification, retinopathy and deafness. Cockayne described syndrome as dwarfism with retinal atrophy and deafness.
Derrnatologic features include photodermatitis with telangiectasia, atrophy, and scarring. The hands and feet are large and cyanotic.
Microcephaly, sunken eyes, severe flexion contractures, dorsal kyphosis, cryptorchidism, cataracts, growth retardation, mental
retardation, hypothalamic and cerebellar dysfunction, and retinitis pigmentosa, optic atrophy may be seen. There is progressive
neurologic disturbance with a shortened lifespan.

Correct Answer. a

Copyright © 2014 Delhi Academy of Medical Sciences, All Rights Reserved. 19/90
(41). Neonatal fat necrosis resembles which of the following

a. Post steroidal panniculitis

b. Erythema induratum

c. Lipodermatosclerosis

d. Lupus panniculitis

Solution. (a) Post steroidal panniculitis


Ref: Read the text below
Sol:
SUBCUTANEOUS FAT NECROSIS OF THE NEWBORN: It is characterized by development of one or more localized, mobile, violaceous,
firm, subcutaneous nodules or plaques during the newborn period of full term neonates. May be associated with hypercalcemia,
thrombocytopenia. Prognosis is fair and histologically shows granulomatous lobular panniculitis with needle shaped clefts in lipocytes or
giant cells. SCLEREMA NEONATORUM is seen within first week in severely ill preterm neonates with widespread, diffuse board like
hardening of skin with livid mottled discoloration and commonly have hypothermia or other medical illness leading to early death in
most. Histology is of sparse inflammation with needle shaped clefts in lipocytes rather than in giant cells and later septal thickening.
POST STEROIDAL PANNICULITIS: It is a complication of rapid systemic steroid withdrawl characterized by subcutaneous nodules on
the cheeks, arms and trunk. Histologically characterized by granulomatous lobular panniculitis with needle shaped clets in lipocytes and
giant cells. The microscopic changes are almost identical to those of subcutaneous fat necrosis of the newborn although calcification and
hemorrhages are more common in the latter. LUPUS PANNICULITIS: LE PROFUNDUS: Presents as tender subcutaneous nodules and
plaques on face, proximal extremities and trunk (breast (lupus mastitis)). The overlying skin may be associated with discoid lupus
erythematosus (LE) in 20% of cases. It is relatively mild, chronic disease mainly affecting females. The lesions begin with subcutaneous
nodules or plaques, where the overlying skin may appear normal, erythematous or may show changes characteristic of discoid LE. The
lesions can be painful and may ulcerate, leading to atrophy and scarring after healing, even giving an anetoderma-like appearance.
Common sites for nodules include thighs, buttocks, arms, breasts and face The major histological changes are lobular panniculitis with
hyaline necrosis of fat and predominantly lymphoplasmacytic infiltrate or lymphoid follicle formation. A lupus band on immunofl
uorescence is positive in most cases, even in the absence of epidermal changes on routine histopathology. NODULAR VASCULITIS
(LOBULAR PANNICULITIS WITH VASCULITIS, ERYTHEMA INDURATUM OF BAZIN AND OF WHITFIELD: Bazin-type if due to
tuberculosis and Whitfield type if not): Nodular vasculitis (NV) is a chronic relapsing lobular panniculitis with septal vasculitis. It is
characterized by a vasculitis of subcutaneous arteries and veins, with subsequent ischaemia of subcutaneous tissue, which results in
clinical suppuration. The cause may be a hypersensitivity reaction to an antigenic trigger including bacterial infections, such as
streptococcal or mycobacterial infections, viral infections such as hepatitis C virus, and drugs such as propylthiouracil autoimmune
colitis etc. It occurs primarily in middle-aged women, and is manifest as tender nodules or plaques on the legs that may later progress to
ulceration. These healthy, middle-aged, sometimes obese, women may have venous stasis. Patients often have ‘thick’ calves with
erythrocyanosis and perifollicular erythema. Histologically, early changes are of leukocytoclastic (sometimes lymphocytic or
granulomatous ) vasculitis of vessels in the subcutaneous tissue leading to ischaemic changes are followed by inflammation and injury to
lipocytes. SCLEROSING PANNICULITIS (LIPODERMATOSCLEROSIS, HYPODERMATITIS SCLERODERMAFORMIS): Describes the
effects in the skin of chronic venous hypertension. Not all cases are overtly associated with chronic venous hypertension Histologically,
there is a combination of fat necrosis and a lobular panniculitis, ultimately leading to fibrous thickening of the fascia and fibrous septae
in the subcutis. Lipomembranous changes of cystic formations and elaborate papillary configurations can coexist. The patients, usually
women in middle age with a high body mass index, gradually develop well-circumscribed, indurated plaques around the lower
extremities, usually but not always on the medial aspect. In acute lipodermatosclerosis the plaques are erythematous, painful and tender,
simulating cellulitis. In the commoner chronic lipodermatosclerosis the indurated areas are skin coloured, with haemosiderin staining,
eventually resulting in fibrotic narrowing of the lower leg.

Correct Answer. a

Copyright © 2014 Delhi Academy of Medical Sciences, All Rights Reserved. 20/90
(42). “Hallucination” is coined by

a. Eugen Bleuler

b. Emil Kraeplin

c. Kurt Schneider

d. Von Domarus

Solution. (a) Eugen Bleuler


Ref: Read the text below
Sol :

Correct Answer. a

Copyright © 2014 Delhi Academy of Medical Sciences, All Rights Reserved. 21/90
(43). All the following groups are vulnerable to enhanced risk of suicide except

a. Widows

b. Positive family history

c. Aggressive behavior

d. Living with parents

Solution. (d) Living with parents


Ref: Read the text below
Sol :

Correct Answer. d

Copyright © 2014 Delhi Academy of Medical Sciences, All Rights Reserved. 22/90
(44). All of the following drugs are employed in the treatment of ADHD except

a. Antidepressants

b. Lithium carbonate

c. Amphetamines

d. Alpha agonists.

Solution. (b) Lithium carbonate


Ref: Read the text below
Sol :
- The characteristic features of (in order of frequency) are :
- Hyperactivity perceptual motor impairment
- Emotional liability, (aggressive outbursts)
- General coordination deficit.
- Attention deficit (distractibility, failure to finish tasks, poor concentration),
- Impulsivity (action before thought, jumping in class, lack of organization).
- Memory & thinking deficit.
- Learning disabilities.
- Speech and hearing deficit.
- Equivocal neurological signs
- EEG abnormalities

Correct Answer. b

(45). Disorientation is commonly seen with

a. Mania

b. Orgaic brain damage

c. Schizophrenia

d. None of theabove

Solution. (b) Orgaic brain damage


Ref: Read the text below
Sol :
- Cognitive or higher mental functions are consciousness, orientation & abstract thinking, are impaired in organic brain syndrome like
dementia, delirium & substance abuse disorders.

Correct Answer. b

Copyright © 2014 Delhi Academy of Medical Sciences, All Rights Reserved. 23/90
(46). CAGE test is for

a. Diagnosing alcoholism

b. Assessing the level of pain

c. Detecting personality disorder

d. Operating conditioning

Solution. (a) Diagnosing alcoholism


Ref: Read the text below
Sol :

Correct Answer. a

(47). Women suffering from postpartum psychiatric disorders would be most likely to demonstrate

a. Marked lability of mood

b. Visual hallucinations

c. Pseudoseizures

d. Grandiose delusions involving the baby

Solution. (a) Marked lability of mood


Reference:Read the text below
Sol:
- Perhaps half or more of all women experience emotional disturbance in the early postpartum period.
- Symptoms usually appear between the third and seventh postpartum day and can include irritability, anxiety,fluctuating mood, feelings
of inadequacy and shame, confusion and disorientation, and such psychotic disturbances as auditory hallucinations and nihilistic
delusions
- Both biologic and psychologic factors appear to be important etiologically in postpartum emotional illness.
- Choice of treatment depends on the type and severity of symptoms.

Correct Answer. a

Copyright © 2014 Delhi Academy of Medical Sciences, All Rights Reserved. 24/90
(48). Tricyclic antidepressant drugs have which of the following pharmacologic properties?

a. Potentiation of the reuptake of norepinephrine

b. Inhibition of noradrenergic function

c. Cholinergic activation

d. Atropinelike activity

Solution. (d) Atropinelike activity


Ref:Read the text below
Sol:
- Tricyclic antidepressants have atropinelike activity as a result of cholinergic blockade and, as a consequence, can produce such
atropinic side effects as dry mouth, blurred vision, urinary retention, and postural hypotension.
- These drugs also enhance noradrenergic functioning by blocking the reuptake of norepinephrine, thus allowing the released
neurotransmitter to stay longer at the receptor site.
- Because the site of antihypertensive action of guanethidine and related drugs is in noradrenergic neurons, drugs like tricyclic
antidepressants that block norepinephrine reuptake will also block uptake of guanethidine, thus inhibiting its effectiveness.

Correct Answer. d

(49). The limbic system of the brain is presumed to be the region most concerned with the operation and expression of emotions. Components
of the limbic system include the

a. Hypothalamus

b. Nucleus accumbens

c. Nucleus of meynert

d. Hippocampus

Solution. (d) Hippocampus


Reference:Read the text below
Sol:
- The limbic system consists of several structures of the brain linked by one common function: control or expression of emotional states
and self-preservation behaviors.
- Specific areas of the brain that constitute the limbic system include the hippocampus and cingulated gyrus (which form part of the
limbic lobe and are presumed to be active in determining emotional response) and the amygdale (which is thought to be involved with
more primitive, instinctual behaviors related to self-preservation).
- The reticular formation is a region of the brain stem regulating arousal and inhibition of the peripheral motor system as well as other
nonvoluntary physiologic functions.

Correct Answer. d

(50). The endocrine abnormalities associated with anorexia nervosa can be described by which of the following statements?

a. Primary pituitary gland dysfunction is the causative mechanism.

b. Amenorrhea usually develops only after weight loss has become significant.

c. Unlike affected women, affected men show little if any endocrine imbalances.

d. Release of gonadotropins is impaired.

Solution. (d) Release of gonadotropins is impaired


Reference:Read the text below
Sol:
- The endocrine disturbances that exist in association with anorexia nervosa arise as a consequence of starvation and thus do not
represent primary dysfunction in the hypothalamus or its target organs, such as the pituitary and adrenal glands.
- Release of gonadotropins is impaired, and secretion activity shows pubertal or even prepubertal diurnal changes.
- Other hormone systems,such as the thyroid, operate abnormally, although they revert to normal when affected persons gain back
weight. Half of affected women developed amenorrhea at the beginning of their weight loss. Men with anorexia nervosa are by no means
immune from hypothalamic dysfunction.

Correct Answer. d

Copyright © 2014 Delhi Academy of Medical Sciences, All Rights Reserved. 25/90
(51). True about incontinenta pigmenti include the following except

a. X-linked dominant

b. Primary skin abnormality

c. Avascularity of peripheral retina

d. Ocular involvement is seen in almost 100% cases and is typically unilateral

Solution. (d) Ocular involvement is seen in almost 100% cases and is typically unilateral
Ref: Rook’s dermatology7th ed.page39.20-39.22
Sol:
- Also known as Bloch-Sulzberger disease, incontinentia pigmenti is an X-linked dominant condition characterized by spattered
pigmentation on the trunk preceded by vesicular and verrucous changes. It appears in girls during the first weeks after birth. Most
lesions are evident by the time the infant is 4 to 6 weeks old. A vesicular phase is present in 87% of cases. This first stage begins in most
individuals before 6 weeks of age and is replaced by verrucous lesions after several weeks to months in two-thirds of patients.
- Although these usually resolve by 1year of age, lesions may persist for many years. In the third or pigmentary phase, pigmented
macules in streaks, sprays, splatters, and whorls follow the lines of Blaschko. The pigmentary stage may last for many years and then
fade away, leaving no sequelae. A fourth stage may be seen in some adult women, manifesting subtle, faint, hypochromic or atrophic
linear lesions, most commonly on the extremties.
- Histologically, the vesicular stage is characterized by spongiosis with eosinophils. As the lesions mature, clusters of dyskeratotic cells
appear within the epidermis. Dyskeratotic cells predominate in the venucous stage, and pigmented incontinence (dermal melanophages)
predominates in hyperpigmented lestons.
- Other cutaneous changes include patchy alopecia at the vertex of the scalp, atrophic changes simulating acrodermatitis chronica
atraphicans on the hands, onychodystrophy, subungual tumors with underlying lytic bone lesions, and palmoplantar hyperhidrosis.
Extracutaneons manifestations occur in 70% to 90% of patients. Most commonly involved are the teeth (up to 90%), bones (40%), central
nervous system (CNS) (33%). and eyes (35%). Immune dysfunction with defective neutrophil chemotaxis and elevated IgE has been
reported. Eosrnophilia is common. Dental abnormalihes usually manifest by the time the individual is 2 years old. Dental defects include
delayed eruption, partial anodontia (43%),microdontia, and cone- or peg-shaped teeth (30%). The most common CNS lindmgs are
seizures mental retardation, spastic paralysis, motor retardation. The eye changes include strabismus, cataracts, retinal detachments,
optic atrophy, blue sclerae, and exudative chorioretinitis. Skeletal abnormalities include syndactyly, skull deformities, dwarfism, spina
bifida, club foot, supernumerary ribs, hemiatrophy, and shortening of the logs and arms. Incontmentia plgmenti is caused by a mutation
in the NEMO gene on the X chromosome, locallzed to Xq28. It is a X-linked dominant trait that is generally lethal for hemizygous affected
males although males with Klinefelter syndrome (47,XXY) may survive. Mosaicism may also account for some cases in males.

Correct Answer. d

(52). Child presents with linear verrucous plaques on the trunk with vacuolization of keratinocytes in S. Spinosum and S. Granulosum.
Diagnosis is ?

a. Incontinenta pigmenti

b. Delayed hypersensitivity reaction

c. Verrucous epidermal nervus

d. Linear darriers disease

Solution. (c) Verrucous epidermal nervus


Ref: Dermatology online journal
Sol:
- Histology of incontinentia pigmenti : the vesicular stage is characterized by spongiosis with eosinophils. As the lesions mature, clusters
of dyskeratotic cells appear within the epidermis. Dyskeratotic cells predominate in the venucous stage, and pigmented incontinence
(dermal melanophages) predominates in hyperpigmented lestons.
- Verrucous epidermal naevus: They are divided into epidermolytic and non-epidermolytic types. In young children they appear as slightly
pigmented velvety or warty streaks or plaques. With age they darken and the surface becomes more warty sometimes with an
erythematous base.
- Epidermolytic and non-epidermolytic verrucous epidermal naevi are sometimes indistinguishable except on histology. Their lack of
inflammation, presence at birth, warty, brown appearance and persistence distinguish them from most other types of epidermal naevus.
- Histology: Epidermolytic verrucous epidermal naevi show hyperkeratosis, acanthosis and papillomatosis with epidermolytic
hyperkeratosis. This comprises perinuclear vacuolization of the keratinocytes, associated with premature and excessive formation of
irregular keratohyalin granules, indistinct cell borders and hyperkeratosis.
- Darier’s diseases is characterized by verrucous malodorous plaques with acantholytic and dyskeratotic keratinocytes (corps ronds and
grains).

Correct Answer. c

Copyright © 2014 Delhi Academy of Medical Sciences, All Rights Reserved. 26/90
(53). Which is false about acrodermatitis enteropathica

a. Triad of diarrhoea, dementia and dermatitis

b. Low serum zinc levels

c. Symptoms improve with zinc supplementation

d. Autosomal recessive

Solution. (a) Triad of diarrhoea, dementia and dermatitis


Ref: Read the text below
Sol:
- Zinc: Deficiency of zinc is associated with a specific dermatitis called ACRODERMATITIS ENTEROPATHICA.
- An autosomal recessive genetic deficiency or abnormality of Zinc Binding Ligands (ZBL) leads to clinical picture of Diarrhea, Alopecia
and Dermatitis (DAD).
- The zinc levels in serum are consistently low. The defect can be overcome by excess oral load of zinc and hence zinc supplementation
improves the symptoms.
- The dermatitis is periorificial & acral and begins as dry, red eczematous patches which soon develop vesicular/pustular changes making
the eruption moist, crusted with large erosions and peripheral scales. (IMP: similar rash in Glucagonoma). It may be associated with
ZEBRA STRIPED Hair

Correct Answer. a

(54). Bullous tinea pedis is most likely caused by which organism?

a. Trichophyton mentagrophytes

b. Trichophyton violaceum

c. Trichophyton soudanese

d. Trichophyton tonsurans

Solution. (a) Trichophyton mentagrophytes


Ref:Read the text below
Sol:
- Trichophyton mentagrophytes is the dermatophyte most commonly associated with inflammatory and bullous tinea pedis, as well as
white superficial onychomycosis in the immunocompetent adult.

Correct Answer. a

(55). Organic cause for erectile dysfunction is most commonly :

a. Psychological

b. Vascular

c. Neuronal

d. Hormonal

Solution. (b) Vascular


Ref: Internet resources
Sol :
- A penile erection is the hydraulic effect of blood entering and being retained in sponge-like bodies within the penis. The process is often
initiated as a result of sexual arousal, when signals are transmitted from the brain to nerves in the penis. Erectile dysfunction is indicated
when an erection is difficult to produce.
- There are various circulatory causes, including alteration of the voltage-gated potassium channel, as in arsenic poisoning from drinking
water.
- The most important organic causes are cardiovascular disease and diabetes, neurological problems (for example, trauma from
prostatectomy surgery), hormonal insufficiencies (hypogonadism) and drug side effects.

Correct Answer. b

Copyright © 2014 Delhi Academy of Medical Sciences, All Rights Reserved. 27/90
(56). In EEG, alpha wave denotes :

a. REM sleep

b. Awake and eye closed and mind wandering

c. Sleep with eyes closed

d. Mental activity

Solution. (b) Awake and eye closed and mind wandering


Ref: Internet resources
Sol :
- Alpha waves are neural oscillations in the frequency range of 8–12 Hz arising from synchronous and coherent (in phase/constructive)
electrical activity of thalamic pacemaker cells in humans. They are also called Berger's wave in memory of the founder of EEG.
- Alpha waves are one type of brain waves detected either by electroencephalography (EEG) or magnetoencephalography (MEG) and
predominantly originate from the occipital lobe during wakeful relaxation with closed eyes. Alpha waves are reduced with open eyes,
drowsiness and sleep.

Correct Answer. b

(57). Counter transference is –

a. Defense mechanism

b. Implies doctor’s feeling towards patients

c. Implies patients feeling towards therapist during psychotherapy

d. Transfer of psychic energy from body parts to brain

Solution. (b) Implies doctor’s feeling towards patients


Ref: Read the text below.
Sol :
Transference-It describes the process of patients unconsciously attributing to their doctors aspects of important past relationship
specially those with their parents. Countertransference-It involves Doctors unconsciously ascribing motives or attributes to patients that
comes from the doctor’s past relationship. Defence mechanism-Most notably used by Sigmund Freud in his psychoanalytic theory, a
defense mechanism is a tactic developed by the ego to protect against anxiety. Defense mechanisms are thought to safeguard the mind
against feelings and thoughts that are too difficult for the conscious mind to cope with. In some instances, defense mechanisms are
thought to keep inappropriate or unwanted thoughts and impulses from entering the conscious mind. In George Eman Vaillant's (1977)
categorization, defences form a continuum related to their psychoanalytical developmental level.[11] Vaillant's levels are:
- Level I - pathological defences (i.e. psychotic denial, delusional projection)
-Level II - immature defences (i.e. fantasy, projection, passive aggression, acting out)
- Level III - neurotic defences (i.e. intellectualization, reaction formation, dissociation, displacement, repression)
- Level IV - mature defences (i.e. humour, sublimation, suppression, altruism, anticipation)

Correct Answer. b

Copyright © 2014 Delhi Academy of Medical Sciences, All Rights Reserved. 28/90
(58). Somatic passivity is seen in –

a. Paranoid schizophrenia

b. Hypochondriasis

c. Depression

d. Body dysmorphic disorder

Solution. (a) Paranoid schizophrenia


Ref: Read the text below.
Sol :

Correct Answer. a

(59). Fear of being in places or situations from which escape might be difficult, embarrassing or in which help may be unavailable in the event
of having panic attack. This condition is called

a. Social phobia

b. Zoophobia

c. Acrophobia

d. Agoraphobia

Solution. (d) Agoraphobia


Ref: Read the text below
Sol :
Agoraphobia
- Most common type of phobia seen in clinical practice; most common in females.
- It is characterized by anxiety about in places or situations from which escape might be difficult or in which help may not be available in
the event of having a panic attack; of being outside the home alone; or of being in public places.
- Depression, depersonalization and obsessional thoughts are more common in agoraphobia than in other phobias.

Correct Answer. d

Copyright © 2014 Delhi Academy of Medical Sciences, All Rights Reserved. 29/90
(60). Which of the following antidepressants is called selective serotonin reuptake inhibitor ?

a. Imipramine

b. Amitriptyline

c. Fluoxetine

d. Desipramine

Solution. -NA-

Correct Answer. c

(61). One of the following is not a feature of dementia

a. Failing memory

b. Impaired judgment

c. Clouding of consciousness

d. Alterations in mood and affect

Solution. (c) Clouding of consciousness


Ref: Read the text below
Sol :
Cardinal features of dementia (chronic organic reaction)
- Strongest risk factor is old age ;
- Global deterioration of higher mental functions in clear consciousness that is progressive (chronic) and (usually) irreversible
- Impaired recent memory (in early stages) short and remote memory (in later stages)
- Deterioration of personality with a lack of personal care
- Impaired judgment and reasoning.

Correct Answer. c

(62). Delirium tremens (DT) is related to

a. Alcohol withdrawal

b. Opioid withdrawal

c. Cocaine withdrawal

d. Cannabis withdrawal

Solution. (a) Alcohol withdrawal


Ref: Niraj Ahuja’s -149, 188.
Sol :
Alcohol withdrawal
- This usually occurs within 1-2 days of abstinence and is characterized by acute tremulousness (earliest and most common feature)
affecting the hands, legs and trunk (‘the shakes) ; nausea; sweating; lability of mood; insomnia and transient hallucinations or illusions
(usually visual). Later there may be epileptic seizures.
- Finally after about 48 hours, delirium tremens may develop – characterized by clouding of consciousness with disorientation in time and
place; visual hallucination (of snakes and bugs) ; tactile hallucinations of insects crawling over the body (formication).

Correct Answer. a

Copyright © 2014 Delhi Academy of Medical Sciences, All Rights Reserved. 30/90
(63). The Act dealing with treatment and welfare of mentally ill persons :-

a. Indian lunacy Act

b. Mental health Act

c. Narcotic drug & Psychotropic substance act

d. Mc. naughten's rule

Solution. (b) Mental health Act


Ref: Ref. A5 - 238
Sol:
- Mental health act came into force in India in 1987. It covers various aspects regarding mentally ill persons. eg.Admission & discharge
from mental hospitals, their rights regarding making will property etc. earlier Indian lunacy act used to govern various aspects of mental
patients.
- Mental Health act 1987 also aims at protection of human rights ill persons.

Correct Answer. b

(64). "Nymphomania" is-

a. Hypoactive sexual desire in female

b. Sexual aversion disorder in male

c. Excessive sexual drive in female

d. Sexual pain syndrome in female

Solution. (c) Excessive sexual drive in female


Ref: Ref. Ahuja 5th ed. - 133
Sol:
- The various sexual desire disorders are hypoactive desire, sexual aversion and excessive sexual drive.
- Excessive sexual drive in female is known as nymphomania while in male is known as Satyriasis, this is a rare variety of sexual
disorders.
- A common variety is hypoactive type which is much more common in females (previously known as frigidity) and it's prevalence
increases with age.

Correct Answer. c

(65). Which of the following would least benefit from treatment with ECT?

a. Severe depression in an elderly person

b. Failure to respond to regular antidepressants

c. Mania

d. Chronic schizophrenia

Solution. (d) Chronic schizophrenia


Reference:Read the text below
Sol:
- ECT has been useful in treating catatonic schizophrenia and some schizophrenias that are fairly new in onset. It has not been useful in
chronic schizophrenia.
- ECT has been most useful in the treatment of affective disorders,namely severe depressions that are debilitating, have severe
vegetative symptoms, have failed to respond to the usual antidepressants, or when there may be severe suicidal symptoms.

Correct Answer. d

Copyright © 2014 Delhi Academy of Medical Sciences, All Rights Reserved. 31/90
(66). Treatment with lithium has been associated with which of the following hormonal changes?

a. Decreased production of TSH

b. Increased release of thyroxine

c. Decreased TSH response to TRH

d. Increase in TSH production

Solution. (d) Increase in TSH production


Ref:Read the text below
Sol:
- Thyroid function studies should be obtained to establish baseline values in a person about to begin lithium therapy.
- Lithium can cause a number of thyroid-related abnormalities, including reduction in thyroxine release,increase in TSH production, and
heightened response of TSH to TRH.
- Lithium can also raise parathyroid hormone levels, although not to clinically significant levels.

Correct Answer. d

(67). Of the following, which is considered a negative symptom of schizophrenia?

a. Anhedonia

b. Loose associations

c. Delusions of thought insertion

d. Incoherence

Solution. (a) Anhedonia


Reference:Read the text below
Sol:
- Negative symptoms of schizophrenia reflect the absence or deficiency of a mental function that is normally present.
- Anhedonia, or the inability to experience pleasure, is an example of such. Positive symptoms of schizophrenia reflect aberrance or
distortion of these mental functions.
- Loose associations,delusions of thought, insertion, incoherence, and stereotypic gestures are all examples of these distortions.

Correct Answer. a

(68). Which of the following medications is appropriate for treating children with attention deficit disorder?

a. Lithium

b. Desipramine

c. Alprazolam

d. Propranolol

Solution. (b) Desipramine


Reference:Read the text below
Sol:
- The antidepressant desipramine has been found effective for treating some cases of attention deficit disorder and offers help to those
children not responsive to the usual treatment with stimulants (methylphenidate,pemoline).
- The remaining choices have not been found useful in treating this condition.
- They are lithium, a mood stabilizer; alprazolam, a benzodiazepine anxiolytic; propranolol, a beta blocker; and perphenazine, an
antipsychotic.

Correct Answer. b

Copyright © 2014 Delhi Academy of Medical Sciences, All Rights Reserved. 32/90
(69). "Seman's technique" is used for treatment of-

a. Impotence

b. Premature ejaculation

c. Sexual aversion disorder

d. None of the above

Solution. (b) Premature ejaculation


Ref: Ref. Ahuja 5th ed. - 137
Sol:
- Seman's or squeeze technique is used in premature ejaculation.
- For impotence Sensate-Focus technique is used, Masters and Johnson's technique is one of the most popular and successful methods of
treatment of psychosexual dysfunctions.
- In Seman's technique, the female partner is asked to manually stimulate the penis causing erection.
- When the male partner experiences 'ejaculating inevitability', the female partner 'squeezes' the penis on the coronal ridge thus delaying
ejaculation.

Correct Answer. b

(70). A 48-year-old woman successfully completes an inpatient program for alcohol detoxification for which she was prescribed
chlordiazepoxide.Upon discharge, the patient is prescribed a medication that disrupts alcohol metabolism. Which of the following
symptoms is the patient likely to experience on taking alcohol?

a. High blood pressure

b. Euphoria

c. Blurred vision

d. Vomiting

Solution. (d) Vomiting


Reference: Read the text below
Sol:
- Ethanol is metabolized in the liver to acetaldehyde, which is enzymatically converted to acetate by the enzyme aldehyde
dehydrogenase.
- Disulfiram is a medication that irreversibly inhibits the action of alcohol dehydrogenase.Acetaldehyde accumulates and accounts for the
aversive effects associated with the disulfiramethanol reaction.
- Flushing, sweating, dyspnea, hyperventilation, tachycardia, hypotension,nausea, and vomiting are common symptoms.
- Extreme reactions can result in respiratory depression, cardiovascular collapse, myocardial infarction, seizures, and death.
- Blurred vision and urinary retention are anticholinergic effects associated with antipsychotic, antidepressant,and antiparkinsonian
medications.
- Hypertension is associated with but not characteristic of the disulfiram-ethanol reaction.
- Disulfiram is not recommended for patients with moderate to severe liver disease, renal failure, severe cardiac disease, pregnancy, or
peripheral neuropathy. In this case, the patient was prescribed chlordiazepoxide and presumably had preserved liver function.

Correct Answer. d

Copyright © 2014 Delhi Academy of Medical Sciences, All Rights Reserved. 33/90
(71). A 22-year-old man with a history of bipolar disorder is prescribed lithium carbonate to treat his symptoms. During a weekend, he hurts
his knee and was prescribed a drug to reduce his symptoms of pain and swelling. Although the patient reports relief from this
medication, he begins to experience abdominal pain, diarrhea, and drowsiness. Which of the following medications may contribute to the
production of these symptoms?

a. Ibuprofen

b. Acetaminophen

c. Aspirin

d. Codeine

Solution. (a) Ibuprofen


Reference: Read the text below
Sol:
- Many nonsteroidal anti-inflammatory drugs can decrease the clearance of lithium and produce significant increases in serum levels.
- Ibuprofen, indomethacin, ketoprofen, diclofenac, phenylbutazone, naproxen, and piroxicam have all been reported to produce such
potential dangerous interactions.
- The symptoms of abdominal pain, diarrhea, and drowsiness indicate mild to moderate (1.5–2.0 mEq/L) lithium toxicity.
- Acetaminophen, aspirin, and the opiates do not interfere with lithium clearance.
- Meperidine, however, can cause dangerous interactions with MAOIs.

Correct Answer. a

(72). Apatient experiences clinical signs and symptoms of depression. The physician decides to prescribe a medication whose mechanism
includes significant blockade of serotonin reuptake. Which of the following antidepressants should be considered?

a. Maprotiline

b. Amitriptyline

c. Bupropion

d. Desipramine

Solution. (b) Amitriptyline


Reference: Read the text below
Sol:
- Clinically effective antidepressant medications potentiate the actions of serotonin, norepinephrine,and dopamine in the brain.
- The antidepressant action of amitriptyline, a TCA, is to significantly block the reuptake of serotonin and to a lesser degree
norepinephrine.
- Bupropion is a weak inhibitor of dopamine reuptake with modest effects on norepinephrine. It does not have any effect on serotonin.
- Maprotiline is a TCA with modest effects on norepinephrine and no effect on serotonin.
- Desipramine, a TCA, has potent effects on norepinephrinereuptake and has weak effects on the serotonergic system.

Correct Answer. b

(73). A patient who is human immunodeficiency virus (HIV)-positive reports a depressed mood, low energy, and hopelessness. Which of the
following medications can the physician prescribe to achieve rapid relief of some of his symptoms?

a. Methylphenidate

b. Fluoxetine

c. Bupropion

d. Buspirone

Solution. (a) Methylphenidate


Reference: Read the text below
Sol:
- Medically ill patients with depressive disorders may respond to psychostimulants, an excellent choice if the patient is unable to tolerate
TCAs.
- The rapid onset of action and rapid clearance are beneficial.
- Nefazodone, fluoxetine, and bupropion are effective antidepressants but may take 4–6 weeks to accomplish beneficial results.
- Buspirone is a nonbenzodiazepine anxiolytic medication not indicated in the treatment of depression.

Correct Answer. a

Copyright © 2014 Delhi Academy of Medical Sciences, All Rights Reserved. 34/90
(74). Drug of choice for treatment and prophylaxis of bipolar affective disorder is :-

a. Lithium

b. Chlorpromazime

c. Haloperidol

d. Diazepam

Solution. (a) Lithium


Reference:Read the text below
Sol:
- Lithium carbonate is the drug used for both treatment of acute phase and prophylaxis against future episode of bipolar affective
disorder.
- Dosage range is 900 mg - 1200 mg/day guided by serum lithium level measurements.

Correct Answer. a

(75). A 4-year-old boy has had a fever for 5 days, cracked lips, conjunctival injection, an exanthem, and cervical lymphadenopathy. What is the
most likely diagnosis?

a. Rubeola

b. Kawasaki disease

c. Erythema infectiosum

d. Rubella

Solution. (b) Kawasaki disease


Ref:Read the text below
Sol:
Kawasaki disease (mucocutaneous lymph node syndrome) is characterized as a fever lasting 5 or more days plus at least four of the
following conditions: (1) acute edema or erythema of the peripheral extremities, followed by desquamation, (2) exanthem, (3) erythema
or cracking of the lips or other changes in the oral cavity (strawberry tongue, pharyngitis), (4) conjunctival injection, (5) acute,
nonpurulent, cervical lymphadenopathy greater than 1.5 cm in diameter (usually unilateral). Treatment includes gamma globulin and
aspirin. Patients should undergo an echocardiogram at baseline and 1 month follow-up.

Correct Answer. b

(76). Regarding the basement membrane zone, decide which of the following is false

a. The lamina lucida separates the trilaminar plasma membrane from the lamina densa.

b. The upper lamina lucida contains the sub-basal cell dense plate.

c. Anchoring fibrils extend from the hemidesmosomes of the basal cell plasma membrane the lamina lucida.

d. Type 4 collagen is found within the lamina densa.

Solution. (c) Anchoring fibrils extend from the hemidesmosomes of the basal cell plasma membrane the lamina lucida.
Ref:Read the text below
Sol:
- Anchoring filaments, rich in laminin 5, run through the lamina lucida and extend from the hemidesmosomes of the basal keratinocyte
cell membrane to the lamina densa

Correct Answer. c

Copyright © 2014 Delhi Academy of Medical Sciences, All Rights Reserved. 35/90
(77). Which of the following is correctly matched:

a. Grattage Test: Pityriasis Rubra Pilaris

b. Buttonholing: neurofibromatosis

c. Carpet tack sign: Systemic lupus erythematosus

d. Apple jelly nodules: Scrofuloderma

Solution. (b) Buttonholing: neurofibromatosis


Ref:Read the text below
Sol:
- Grattage test & Auspitz's sign is positive when slight scratching or curetting of a scaly lesion reveals initially fine candle wax scales
followed by red Berkley’s memb. This then gets removed to reveal punctate bleeding points(Auspitz’s sign) within the lesion which
suggests of psoriasis. Carpet
- Tack Sign: or carpet en tack sign: removal of adherent scales in DLE reveals downward projection of scales which are follicular plugs.
- Buttonholing sign: is seen in Neurofiibromatosis 1 lesions of skin where when pressed at the centre of lesion the whole lesion
disappears under the skin as if has passed through the buttonhole. Pityriasis rubra pilaris can have characteristic

Correct Answer. b

(78). All are true for Tuberous sclerosis except

a. Hypomelanotic macules

b. Shagreen patch

c. Koenens tumors

d. Autosomal recessive

Solution. (d) Autosomal recessive


Ref:Read the text below
Sol:
- Tuberous sclerosis complex (TSC)is an autosomal dominant disease arising from a genetically programmed hyperplasia of ectodermal
and mesodermal cells and manifested by a variety of lesions in the skin, CNS (hamartomas),heart, kidney, and other organs.
- The principal early manifestations are the Vogt’s triad of seizures, mental retardation, and congenital white spots (macules). Facial
angiofibromata are pathognomonic but do not appear until the third or fourth year.
- Skin lesions include Hypomelanotic Macules present at birth in _80% of patients—“ Polygonal” or “thumbprint,” lance ovate or
“ashleaf” spots, or tiny white macules or “confetti,”. Papules/Nodules .1 to .5 cm, dome-shaped, firm and smooth, exhibiting red or skin
color Occurring in the center of the face termed adenoma sebaceum but represent angiofibromas (present in 70%).
- Plaques Represent connective tissue nevi (“shagreen” patch), present in 40%; skin colored; occur on the back and buttocks. Periungual
Papules or Nodules Ungual fibromas (Koenen’s tumors) present in 22%, arise late in childhood and have the same pathology
(angiofibroma) as the facial papules.
- CNS (Subependymal nodule, Subependymal giant cell astrocytoma), eye (gray or yellow retinal plaques, 50%), heart (benign
rhabdomyomas), hamartomas of mixed-cell type (kidney, liver, thyroid, testes, and GI system esp. renal angiomyoliomas and pulmonary
lymphangioleiomyomatosis.) are systemic involvements in TSC.

Correct Answer. d

(79). Wornoff’s ring is associated with

a. Psoriasis

b. Vitiligo

c. Lichen planus

d. Tuberous sclerosis

Solution. (a) Psoriasis


Ref:Read the text below
Sol:
- Wornoff’s ring is seen in psoriasis when lesions are tending to heal.It is prostaglandin mediated vascular phenomenon and no defect of
melanin is seen.

Correct Answer. a

Copyright © 2014 Delhi Academy of Medical Sciences, All Rights Reserved. 36/90
(80). Regarding vitillgo all are true except

a. Genetic predisposition

b. Steroids used in localized disease

c. Leukotrichia has a good prognosis

d. Narrow band UVB used in treatment

Solution. (c) Leukotrichia has a good prognosis


Ref:Read the text below
Sol:
- Vitiligo is an idiopathic dermatoses with strong genetic influences leading to an autoimmune destruction of melanocytes in the skin
which produces depigmented patches characterstic of vitiligo.
- They may be associated with white hair (poliosis or leukotrichia) within the vitiliginous patches. As they are important source of
repigmentation on treatment the loss of hair follicular pigmentation indicates a poorer prognsis.
- The treatment options are use of topical steroids and tacrolimus in limited cases, PUVA, Narrow band UVB (311nm) etc.
- In more aggressive variety systemic steroids in pulse form may be used. A multimodal approach generally gives better response.

Correct Answer. c

(81). Which of the following is the most common type of psoriasis ?

a. Guttate psoriasis

b. Pustular psoriasis

c. Stable plaque psoriasis

d. Erythrodermic psoriasis

Solution. (c) Stable plaque psoriasis


Ref.: Harrison’s- 316
Sol :

Correct Answer. c

Copyright © 2014 Delhi Academy of Medical Sciences, All Rights Reserved. 37/90
(82). The infectivity of a convalescent case of cholera lasts for :

a. Less than 7 days

b. 7 to 14 days

c. 14 to 21 days

d. 21 to 28 days

Solution. (c) 14 to 21 days


Ref: Read the text below
Sol :

Correct Answer. c

(83). The syndromic management of genital ulcer syndrome in India includes which of the following diseases?

a. Chancroid and primary chancre

b. Chancroid and herpes simplex

c. Chancroid, primary chancre and herpes simplex

d. Herpes simplex and primary chancre

Solution. (c) Chancroid, primary chancre and herpes simplex


Ref:
Neena Khanna-270-72
Sol :
Due to paucity of available investigations in the field set- up coupled with difficulties in reaching an accurate diagnosis by health
workers, the national programme has suggested classification and treatment of STI’s on the basis of a syndromic approach (A non-
etiologic diagnosis model based only on the predominant presenting symptom) as follows:
1) Genital ulcer disease (GUD)
2) Genital discharges- Urethral discharge (UD), anorectal discharge (ARD), vaginitis, cervicitis
3) Genital growths
4) Genital swellings
5) Inguinal bubo (IB)
6) Lower abdominal pain (LAP)

Correct Answer. c

Copyright © 2014 Delhi Academy of Medical Sciences, All Rights Reserved. 38/90
(84). A patient gives a h/o recurrent oral ulcers. The ulcers are small with a yellow floor surrounded by an erythematous halo on the lips. He
also has multiple, tender nodules on the shin. He most likely has –

a. Behcet’s syndrome

b. Herpes labialis

c. Fixed drug eruption

d. Pemphigus vulgaris

Solution. (a) Behcet’s syndrome


Ref: NEENA KHANNA -202-03
Sol :
Behcet’s syndrome-Behcet’s syndrome is an disorder of unknown etiology having strong association with HLA B51. A possibility that
infectious agents (e.g. streptococci) trigger an immunoregulatory defect in genetically predisposed individuals is an attractive
possibility.Its more frequently seen in age group 20-30 years having male preponderance.
Clinical features- Its characterized by aphthous ulcers which are recurrent numerous and large.Ulcers have a necrotic floor surrounded
by halo of erythema.Ulcer are present on oral and genital mucosae. Other features include erythema nodosum like lesions(which are
deep seated multiple tendor nodules on shin area), palpable purpuric lesions, and cutaneous pustular vasculitic lesions. ( pathergy test-
Its used for diagnosis of Behcet’s syndrome.Positive pathergy performed with oblique insertion of a 20 guage needle and read by
physician at 24-48 hours.) Fixed drug eruption- these are characterized by well defined circular deeply erythematous plaques which
sometimes develop central bullae.They subside leaving behind slate gray hyperpigmentation which persist btwn acute episodes.Lesions
recur at same site each time the offending drug is taken usully after 8-16 hours after intake of drug. Drugs implicated are-
phenophthelin(present in purgatives), barbiturates, sulphonamides, tetracyclines, salicylates and phenacetin. Pemphigus vulgaris It is
characterized by flaccid bullae appearing o normal skin which rupture to form painful erosions which have tendency to spread and take
very long to re-epethialise. Nikolsky sign and Bulla spread sign are positive. IN 50% pts lesions begin in oral mucosa.eventually all pts
develop mucosal lesions.oral mucosa is involved most frequently.Painful erosions which extend peripherally with shedding of the mucosa.
Herpes labialis- It is caused by HSV hominis type1.(HSV hominis has two antigenic types 1 and 2. Type 2 causes genital infection). HSV
type1 infection is characterized by Acute gingivostomatitis(closely grouped vesicles which rapidly form polycyclic ulcers with a yellow
psuedomembrane and heal in about a fortnight.). Malaise fever and lymphadenopathy are frequent.

Correct Answer. a

Copyright © 2014 Delhi Academy of Medical Sciences, All Rights Reserved. 39/90
(85). The only definitive indication of systemic corticosteroids in psoriasis is –

a. Pustular psoriasis

b. Impetigo herpetiformis

c. Erythroderma

d. Psoriatic arthropathy

Solution. (b) Impetigo herpetiformis


Ref: Neena khanna -202-03
Sol :
Corticosteroids in psoriasis- Monitored use of topical corticosterois in limited stable plaque psoriasis is a good therapeutic option bcause of ease of
application and removal, lack of irritation and absence of staining of skin or linen. But long term use of corticosteroid can result in dermal
atrophy,tachyphylaxis,early relapses,precipitation of unstable psoriasis and rarely when used in extensive lesions,adrenal suppression(due systemic
absorption of steroids). Use of systemic corticosteroids should be avoided at all costs except in pustular psoriasis in pregnancy which is also known
as impetigo herpetiformis. Pustular psoriasis can be divided into localized and generalized forms

Correct Answer. b

Copyright © 2014 Delhi Academy of Medical Sciences, All Rights Reserved. 40/90
(86). Which of the following is true about somatization disorder?

a. Having a biological parent with antisocial personality disorder has no correlation with risk of having a somatization disorder.

b. Symptoms typically begin before age 30.

c. Symptoms involve a single organ system.

d. It generally remits within 6 months of onset.

Solution. (b) Symptoms typically begin before age 30.


Ref:Read the text below
Sol:
- Typically, somatization symptoms begin before age 30 and occur over several years.The symptoms tend to fluctuate but rarely
completely remit.
- Studies show that both genetic and environmental factors contribute to the risk of developing this disorder.
- Additionally, family studies show a correlation among antisocial personality disorders, substance- related disorders, and somatization
disorders.
- The prevalence rate for women ranges from 0.2 to 2%.

Correct Answer. b

(87). A 34-year-old man reveals that he derives sexual satisfaction from rubbing up against women on crowded subway trains. Which of the
paraphilias is this an example of?

a. Exhibitionism

b. Pedophilia

c. Voyeurism

d. Frotteurism

Solution. (d) Frotteurism


Reference: Read the text below
Sol:
- Paraphilias encompass a wide variety of maladaptive sexual behaviors and fantasies.
- Frotteurism is the syndrome of recurrent,intense sexual fantasies and behaviors involving touching or rubbing against a nonconsenting
adult.
- Exhibitionism involves revealing one’s genitals to unsuspecting strangers.Pedophilia is sexual fantasies and behaviors involving
children.
- Voyeurism involves secretly watching someone involved in disrobing, nudity, or sexual behavior. Fetishism is a sexual attraction to an
inanimate object.

Correct Answer. d

(88). After being severely reprimanded by his employer, a man goes home and is extremely nasty to his wife. What is his behavior an example
of ?

a. Sublimation

b. Dissociation

c. Displacement

d. Rationalization

Solution. (c) Displacement


Reference: Read the text below
Sol:
- The man is naturally angry, anxious, and sensitive at being reprimanded by his employer. He has found it difficult to express his feelings
toward the disturbing person, the employer.
- Instead of suppressing or repressing the anger, or sublimating his tension in more forceful work, he displaces his anger onto a safer
target, his wife. This is an example of displacement.

Correct Answer. c

Copyright © 2014 Delhi Academy of Medical Sciences, All Rights Reserved. 41/90
(89). Which of the following relations is least likely to exist?

a. Crime and substance abuse

b. Trauma and alcohol abuse

c. Cocaine and sexually transmitted diseases

d. Intravenous drug use and long life expectancy

Solution. (d) Intravenous drug use and long life expectancy


Reference: Read the text below
Sol:
- Illicit substance users are 18 times more likely to be involved in criminal activities. Elevated serum alcohol levels are found in at least
35 percent of trauma patients.
- Mental health patients have an increased frequency of suicidal ideation and gestures.
- Intravenous drug users have an increased risk of intentional and accidental overdose, infection, and blood-borne diseases, thereby
decreasing their normal life expectancy.
- Crime, substance abuse, trauma, and medical problems are all interrelated and underscore the need for physicians to be proactive in
health education and legislative activities.

Correct Answer. d

(90). A false belief unexplained by reality, shared by a number of people is :-

a. Delusion

b. Obsession

c. Superstition

d. Hallucination

Solution. (c) Superstition


Reference:Read the text below
Sol:
- Superstition is a false belief, unexplained by reality & shared by a number of people.
- Delusion is a false belief based on incorrect inference about external reality, not consistent with patients intelligence & cultural
background. Cannot be corrected by reasoning.
- Obsession is pathological persistence of an irrestible thought or feeling that cannot be eliminated from consciousness by logical effort,
associated with anxiety.
- Hallucination is a perceptual disorder

Correct Answer. c

(91). One of the following in not a pervasive developmental disorder

a. Autistic disorder

b. Conduct disorder

c. Asperger’s disorder

d. Rett’s disorder

Solution. (b) Conduct disorder


Ref: Niraj - 176
Sol :

Correct Answer. b

Copyright © 2014 Delhi Academy of Medical Sciences, All Rights Reserved. 42/90
(92). Most common hallucinations that occur in schizophrenia are

a. Visual hallucinations

b. Tactile hallucinations

c. Auditory hallucinations

d. OIfactory hallucinations

Solution. (c) Auditory hallucinations


Ref: Niraj - 60
Sol :
- Hallucinations are common in schizophrenia,auditory hallucinations are most common.
- Third person hallucinations are characteristic (e.g voices heard arguing’, discussing the patient in the third person).
- Thought echo – audible thoughts.
- Elementary auditory hallucinations : hearing simple sounds rather than voices.
- Voices commenting on one’s action.

Correct Answer. c

(93). Type D personality has been recently found to be a risk factor for -

a. Coronary artery disease

b. Depression

c. Personality disorder

d. Schizophrenia

Solution. (a) Coronary artery disease


Ref: http://www.health.harvard.edu/press_releases/type_d_personality Sol :
Research has shown that CHD patients with a Type D personality have a worse prognosis following a myocardial infarction (MI) as
compared to patients without a Type D personality. Type D is associated with a 4-fold increased risk of mortality, recurrent MI, or sudden
cardiac death, independently of traditional risk factors, such as disease severity. People with a set of traits known as the Type D
(“distressed”) personality suffer from a high degree of emotional distress, but they consciously suppress their feelings. Early studies
show that once Type D’s develop coronary artery disease, they are at greater risk of dying, and they often have a poorer quality of life.
Personality types - Type A and Type B personality theory. According to this impatient, achievement-oriented people are classified as Type
A, whereas easy-going, relaxed individuals are designated as Type B. The theory originally suggested that Type A individuals were more
at risk for coronary heart disease, but this claim has not been supported by empirical research.

Correct Answer. a

(94). Which of the following is not a specific somatoform disorder-

a. Chronic fatigue syndrome

b. IBS

c. Fibromyalgia

d. Somatization disorder

Solution. (c) Fibromyalgia


Ref: http://www.emedicinehealth.com Sol : Fibromyalgia has organic basis behind its pathology.Rest all options are without any known
pathology. Somatoform disorder- 1.Somatization disorder 2.Conversion disorder 3.Hypochondriasis 4.Factitious Disorder 5.Somatoform
Pain disorder Fibromyalgia: While there is no known cause for fibromyalgia, recent research has revealed some new facts about the
disease. One of the new discoveries is that people with fibromyalgia process pain differently. The level of chemical in the cerebrospinal
fluid(CSF) called substance P, which transmits pain impulses to the brain, is three times higher in people with the disease than in those
who do not have the condition. This likely causes someone with fibromyalgia to experience pain more intensely Chronic fatigue syndrome
(also called CFS)- is a disorder without a known cause, although CFS may be related to a previous infection. CFS is a state of chronic
fatigue that exists without other explanation for six months or more and is accompanied by cognitive difficulties (problems with short-
term memory or concentration).
IBS:Irritable bowel syndrome (IBS) is a chronic gastrointestinal disorder of unknown cause. Common symptoms include abdominal
cramping or pain, bloating and gassiness, and altered bowel habits.

Correct Answer. c

Copyright © 2014 Delhi Academy of Medical Sciences, All Rights Reserved. 43/90
(95). Most severe manifestation of alcohol withdrawl syndrome :

a. Wernicke’s encephalopathy

b. Delirium tremens

c. Seizures

d. Aloholic hallucinosis.

Solution. (b) Delirium tremens


Ref: Read the text below
Sol :
- Delirium tremens (DT) is the most severe form of ethanol withdrawal manifested by altered mental status and sympathetic overdrive,
which can progress to cardiovascular collapse.

Correct Answer. b

(96). Most common type of schizophrenia is:

a. Paranoid

b. Simple

c. Catatonic

d. Undifferentiated

Solution. (a)Paranoid
Ref: Read the text below
Sol :
- The different types of schizophrenia are based on the specific symptoms a person is experiencing.
- Since the symptoms of schizophrenia can change over time, it is possible for a person to have more than one type during his or her
lifetime.
- Schizophrenia types include :
i. Paranoid schizophrenia.
ii. Disorganized (hebephrenic) schizophrenia.
iii. Catatonic schizophrenia.
iv. Residual and latent schizophrenia.
v. Undifferentiated disorder-this is a very common type of schizophrenia.
vi. Simple schizophrenia.
vii. Post-schizophrenic depression
viii. Others.

Correct Answer. a

Copyright © 2014 Delhi Academy of Medical Sciences, All Rights Reserved. 44/90
(97). Inter cellular IgG deposition in epidermis is seen in

a. Pemphigus

b. Sub corneal pustular dermatosis

c. Bullous pemphigoid

d. Dermatitis herpetiformis

Solution. (a) Pemphigus


Ref: Neena Khanna -62
Sol :

Correct Answer. a

(98)
Granular IgA deposit at dermal papilla are found in
.

a. Dermatitis herpetiformis

b. IgA disease of childhood

c. Herpetic gestation

d. Bullous pemphigoid

Solution. (a) Dermatitis herpetiformis


Ref: Neena Khanna - 62
Sol :

Correct Answer. a

Copyright © 2014 Delhi Academy of Medical Sciences, All Rights Reserved. 45/90
(99). A 3 year old child has eczematous dermatitis on extensor surfaces.His mother has a history of bronchial asthma.Diagnosis should be-

a. Atopic dermatitis

b. Contact dermatitis

c. Seborrhic dermatitis

d. Infantile eczematous dermatitis

Solution. (a) Atopic dermatitis


Ref: Roxburgh -105-112
Sol :
- Eczematous dermatitis on extensor surface and the history of bronchial asthma suggests the dianosis of atopic dermatitis
Atopic dermatitis Etiology and pathogenesis
- Genetic predisposition is very important but the precise mode of inheritance is uncertain.
- Atopic disease tend to run true to type within each family. In some families the affected members predominantly have eczema while in
others respiratory problems predominate.
This is probably beacuase the dermatitis and asthma are inherited through separate though closely related genetic pathways.
Immunological changes
- Elevated IgE level seen in 80% of the patients
- Abnormalities in lymphocytes are also seen
Clinical features Clinical picture varies with the age of the patient (i) infantile
- Begins at about 3 months
- Severely itchy exudative lesions on face and other parts
(ii) childhood
- Itch leathery flexural lesions, sometime extensor lesions occur
(iii) Adult
- Lichnefied itchy lesions
- Seen in cubital and popliteal fossa
Seborrhic dermatitis
- Caused by an yeast, malassezia furfur
- Usually seen in adults, sometimes in infants but not in children
- The lesion involve the scalp, face (nasolabial folds, eyebrowns and eyelashes) presternal and interscapular regions and major flexures
(axilla, groin and inframammary region) umbilicus and natal cleft. This distribution of seborrhic dermatitis is very characteristic and is
called ‘seborrhic districbution’
Contact dermatitis
- It develops due to contact with chemicals.

Correct Answer. a

Copyright © 2014 Delhi Academy of Medical Sciences, All Rights Reserved. 46/90
(100 The standard regimen for satisfactory treatment of paucibacillary
). leprosy consists of :

a. Dapsone alone for 12 months

b. Dapsone and rifampicin for 6 months

c. Dapsone and clofazimine for 6 months

d. Dapsone and ethionamide for 12 months

Solution. (b) Dapsone and rifampicin for 6 months


Ref: Park - 261
Sol :

Correct Answer. b

(101). A child presented with complaint of severe itching over the web of fingers, more at night. Examination revealed burrows. Most probable
diagnosis is :

a. Tinea cruris

b. Scabies

c. Infantile eczema

d. Papular urticaria.

Solution. (b) Scabies


Ref: Neena -294, 295
Sol :
- Scabies is essentially a disease of the children.
- The itching appears a few days after infestation.
- It may occur within a few hours if the mite is caught a second time.
- The itch is characteristically more severe at night and affects the trunk and limbs.
- It does not usually affect the scalp.
- Burrow is the path gnomic lesion of scabies.
- Scabies burrows appear as tiny grey irregular tracks between the fingers and on the wrists.
- They may also be found in armpits, buttocks, on the penis, insteps and backs of the heels.
- Microscopic examination of the contents of a burrow may reveal mites, eggs or mite faeces (scybala).

Correct Answer. b

Copyright © 2014 Delhi Academy of Medical Sciences, All Rights Reserved. 47/90
(102). Air-borne contact dermatitis can be diagnosed by:

a. Skin biopsy

b. Patch test

c. Prick test

d. Estimation of serum IgE levels

Solution. (b) Patch test


Ref:Read the text below
Sol:
- Air-borne contact dermatitis (ABCD) is a type of allergic contact dermatitis (ACD). ACD is confirmed with a patch testing as it detects a
type-IV hypersensitivity reaction to exogenous allergens.
- Parthenium hysterophorous (congress grass) is the commonest allergen causing ABCD in India.
- Skin biopsy will only identify a dermatitis but there are no specific changes to any dermatitis and it is the clinical scenario that will lead
to a diagnosis.
- Prick-test is to identify immediate type hypersensitivity (type-I), while serum IgE is a marker of a hypersensitive state seen in atopic
dermatitis, asthma,parasitic infestations and hypereosiniophilia syndromes.

Correct Answer. b

(103). Progression to abscess formation in nerves is most commonly seen in

a. Tuberculoid leprosy

b. Borderline lepromatous leprosy

c. Lepromatous leprosy

d. Borderline leprosy

Solution. (a) Tuberculoid leprosy


Ref: Harrison-1024
Sol :
- Patients with various forms of leprosy, but particularly those with the Borderline Tuberculoid (BT) form, may develop abscesses of
nerves (most commonly the ulnar) with an adjacent cellulitic appearance of the skin.
- In such conditions, the affected nerve is swollen and exquisitely tender.
- Although glucocorticoids may reduce signs of inflammation, rapid surgical decompression is necessary to prevent irreversible sequelae.

Correct Answer. a

(104). Jock itch is caused by

a. Tinea pedis

b. Tinea capitis

c. Tinea unguium

d. Tinea cruris

Solution. (d) Tinea cruris


Ref: Read the text below
Sol :
Jock itch, also called tinea cruris or ringworm of the groin, is an infection of the groin area caused by fungus. Symptoms
- Itching in groin, thigh skin folds, or anus
- Red, raised, scaly patches that may blister and ooze -- The patches often have sharply-defined edges and are often redder around the
outside with normal skin tone in the center
- Abnormally dark or light skin

Correct Answer. d

Copyright © 2014 Delhi Academy of Medical Sciences, All Rights Reserved. 48/90
(105). Erysipelas is a skin infection often caused by

a. Erysipelothrix rhusiopathiae

b. Group A beta-hemolytic streptococci

c. Trichophyton rubrum

d. Pseudomonas aeroginosa

Solution. (b) Group A beta-hemolytic streptococci


Ref: Harrison-800
Sol :

Correct Answer. b

Copyright © 2014 Delhi Academy of Medical Sciences, All Rights Reserved. 49/90
(106
All of the following are true about bullous impetigo except
).

a. Caused by Staphylococcus aureus

b. Common in neonates and infants

c. Lesions commonly occur on face

d. Lesions have an erythematous base

Solution. (d) Lesions have an erythematous base


Ref: OP Ghai-658
Sol :

Correct Answer. d

(107). Infectious cause of erythematous rashes is

a. Measles

b. Typhoid fever

c. Yellow fever

d. Malaria

Solution. (a) Measles


Ref: Harrison- 128
Sol :
- The rash of rubeola (measles) starts at the hairline 2-3 days into the illness and moves down the body, sparing the palms and soles. It
begins as discrete erythematous lesions, which become confluent as the rash spreads.
- Koplik’s spots (1 to 2-mm white or bluish lesions with an erythematous halo on the buccal mucosa) are path gnomonic for measles and
are generally seen during the first 2 days of symptoms.

Correct Answer. a

Copyright © 2014 Delhi Academy of Medical Sciences, All Rights Reserved. 50/90
(108). Hemidesomsome is located in the

a. Horny cell layer

b. Granular cell layer

c. Squamous cell layer

d. Basal Cell layer

Solution. (d) Basal Cell layer


Ref: Clinical dermatology- 12.
Sol :
Hemidesmosomes :
- Are very small stud-or rivet-like structures on the inner basal surface of basal cells of epidermis of skin.
- Connect the basal face to other cells.
- Use integrin cell adhesion proteins. Attach one cell to the extracellular matrix.
- Contain collagen 17, alpha 6 and beta 4 integrins, bullous pemhpigoid coantigens.

Correct Answer. d

(109). Estimation of serum levels is important for which of the following drug

a. Haloperidol

b. Benzodiazepines

c. Lithium

d. Chlorpromazine

Solution. (c) Lithium


Ref: Read the text below
Sol :
- Lithium has been the drug of choice for the treatment of maniac episode (acute phase) as well as for prevention of further episodes in
bipolar mood disorder.
- It has also been used in the treatment of depression with less success.
- Lithium needs to be closely monitored by repeated blood levels, as the difference between the therapeutic (0.8-1.2 mEq/L) and lethal
blood levels (more than 2.5-3.0 mEq/I) is not very wide (narrow therapeutic index)

Correct Answer. c

(110). Drug of choice for attention deficit disorder with hyperactivity is :

a. Chlorpromazine

b. Clonidine

c. Imipramine

d. Methylphenidate

Solution. (d) Methylphenidate


Ref: Read the text below
Sol :
- Attention – Deficit Hyperactivity Disorder/ADHD/AD/HD is a neurobehavioral developmental disorder.
- It is primarily characterized by “the co-existence of attentional problems and hyperactivity, with each behavior occurring infrequently
alone” and symptoms starting before seven years of age.
- ADHD is the most commonly studied and diagnosed psychiatric disorder in children, affecting about 3% to 5% of children globally.
- Stimulant medication are the most commonly prescribed medications for ADHD.
- The most common stimulant medications are the chain substituted amphetamine methylphenidate (currently, the drug of choice),
dextroamphetamine and dexamohetamine.
- Atomoxetine, a norepinephrine reuptake inhibitor, may be an alternative fore children who do not respond to stimulants.

Correct Answer. d

Copyright © 2014 Delhi Academy of Medical Sciences, All Rights Reserved. 51/90
(111). Which of the following is a shortest acting benzodiazepine :

a. Diazepam

b. Lorezepam

c. Midazolam

d. Alprazolam

Solution. (c) Midazolam


Ref: Read the text below
Sol :
- A benzodiazepine can be placed into one of three groups by its elimination half-life, or time it takes for the body to eliminate half of the
dose.
- Some benzodiazepines have long-acting active metabolites such as diazepam and chlordiazepoxide, which are metabolized into
desmethyldiazepam. Desmethyldiazepam has a half-life of 36-200 hours.
- Very short-acting compounds have a half-life of 2-5 hours.
They have few residual effects if taken before bedtime, rebound insomnia may occur upon discontinuation and they might cause day time
withdrawal symptoms such as next day rebound anxiety with prolonged usage. Examples are brotizolam, midazolam and triazolam.

Correct Answer. c

(112). Illusion is :

a. A false unshaken belief not keeping one’s sociaocultural background

b. Perception without stimuli

c. Abnormal perception by a sensory misinterpretation of actual stimulus

d. Fear of closed spaces.

Solution. (c) Abnormal perception by a sensory misinterpretation of actual stimulus Ref:


Internet resources
Sol :
- Illusion is an abnormal perception caused by a sensory misinterpretation of actual stimulus, sometimes precipitated by strong emotion,
e.g. fear provoking a person to imagine they have seen an intruder in the shadows.

Correct Answer. c

(113). The term psychoanalysis is associated with :

a. John Broadus Watson

b. Carl Gustav Jung

c. Sigmond Freud

d. Wilhelm Reich

Solution. (c) Sigmond Freud


Ref: Read the text below
Sol :
- Psychoanalysis/Freudian psychology is a body of ideas developed by Austrian physician Sigmund freud and continued by others.
- It is primarily devoted to the study of human psychological functioning and behavior, although it can also be applied to societies.
- Psychoanalysis has three applications :
i. A method of investigation of the mind and the way one thinks;
ii. A systematized set of theories about human behavior.
iii. A method of treatment of psychological or emotional illness.

Correct Answer. c

Copyright © 2014 Delhi Academy of Medical Sciences, All Rights Reserved. 52/90
(114). A child psychiatrist would like to evaluate the intellectual ability of a 3-year-old patient. Which of the following is the most appropriate
test for him to use?

a. WAIS-R

b. WISC III

c. Denver Developmental Scale

d. Stanford-Binet Scale

Solution. (d) Stanford-Binet Scale


Ref:Read the text below
Sol:
- The Stanford-Binet scale is best for younger children (2-4 years old), since it does not rely exclusively on language.
- The Denver Developmental Scale is used to assess the attainment of developmental milestones in children under 2.
- The WAIS-R (Wechsler Adult Intelligence Scale; is used for individuals aged 17 and over. (Just think, the WAIS-R is rated "R").
- The WISC III (Wechsler Intelligence Scale for Children; is useful for evaluating children aged 6-16.
- The WPPSI (Wechsler Preschool and Primary Scale of Intelligence; is used for children aged 4-6.

Correct Answer. d

(115). A lady, while driving a car meets with an accident. She was admitted in an ICU for 6 months. After being discharged, she often gets up in
night and feels terrified. She is afraid to sit in car again. The diagnosis is

a. Panic disorder

b. Phobia

c. Conversion disorder

d. Post traumatic stress disorder

Solution. (d) Post traumatic stress disorder


Ref:Read the text below
Sol:
- Post-traumatic stress disorder (PTSD) may occur soon after a major trauma, or it can be delayed for more than 6 months after the
event.
-When it occurs soon after the trauma, it usually gets better after 3 months. However, some people have a longer-term form of PTSD,
which can last for many years.
- PTSD can occur at any age and can follow a natural disaster such as a flood or fire, or events such as war, a prison stay, assault,
domestic abuse, or rape

Correct Answer. d

(116). A patient present with waxy flexibility, negativitism and rigidity. Diagnosis is

a. Catatonic Schizophrenia

b. Paranoid schizophrenia

c. Hebephrenic schizophrenia

d. Simple schizophrenia

Solution. (a) Catatonic Schizophrenia


Ref:Read the text below
Sol:
Catatonic schizophrenia is characterized by marked disturbance of motor behaviour in addition to the general features of schizophrenia.
The patient in question present with typical signs of retarded catatonia, and hence is catatonic schizophrenia.
Certain characteristics catatonic sings often in our examinations include: 1. Mutism : absence of speech 2. Rigidity: Maintenance of Rigid
posture against efforts to be moved 3. Posturism: Voluntary assumption of bizarre, inappropriate posture for long period of Time 4.
Negativis : Motivates resistance to all commands and attempts to be moved 5. Echolalia: Repetition, echo or mimicking of phrases of
words heard 6. Echopraxia : Repetition, echo or mimicking of actions observed

Correct Answer. a

Copyright © 2014 Delhi Academy of Medical Sciences, All Rights Reserved. 53/90
(117). Chandu, a age 32 presents with abdominal pain and vomiting. He also complains of some psychiatric symptoms and visual
hallucianations. Most likely diagnosis is

a. Intermittent, porphyria

b. Hypothyroidism

c. Hyperthyroidism

d. Hysteria

Solution. (a) Intermittent, porphyria


Ref:Read the text below
Sol:
- Acute intermittent porphyria is a neurovisceral syndrome, and should be suspected whenever a patient presents with a constellation of
symptoms, suggestions involvement of both viscera and nervous system.
- The patient in question is presenting with symptomatology that is consistent with a diagnosis of acute intermittent porphyria.

Correct Answer. a

(118). A 37-year-old woman who was diagnosed with AIDS 3 years earlier is unable to work, is physically debilitated, and requests her physician
to provide her with medications with which to take her own life. The most common emotional disorder that results in such requests by
patients is

a. Schizophrenic disorder

b. Factitious disorder

c. Major depressive disorder

d. Borderline personality disorder

Solution. (c) Major depressive disorder


Ref:Read the text below
Sol:
- Many people who request physician-assisted suicide have one of two conditions present: either a poorly controlled painful condition or
severe depression.
- If the painful condition is adequately treated or the depression is brought under good medical control, the request for physician
assistance in terminating the situation is typically withdrawn.
- It is important to note that bringing these conditions under control requires the intervention of caregivers who are specifically trained
in the management of these two conditions; primary care physicians usually are not adequately trained to address these difficult
presentations.
- Patients who are diagnosed as bipolar disorder borderline personality disorder), and schizophrenic disorder often make suicide
attempts (and frequently complete those attempts), they do not generally ask their physician for assistance in the suicide.
- Persons with factitious disorder are seeking primary gain, often for dependency needs, and are seeking to enter the "sick role" not the
"dead role.

Correct Answer. c

(119). Ability to filter unpleasant memories is called

a. Dissociation

b. Repression

c. Suppression

d. Isolation

Solution. (b) Repression


Ref:Read the text below
Sol:
Repression may refer to:
- Memory inhibition, the ability to filter irrelevant memories from attempts to recall
- Political repression, the oppression or persecution of an individual or group for political reasons
- Social repression
- Psychological repression, the psychological act of excluding desires and impulses from one's consciousness
- Repression, a theoretical concept that describes the act of creating a repressed memory that excludes a particular episode from
consciousness and the ability to recall it

Correct Answer. b

Copyright © 2014 Delhi Academy of Medical Sciences, All Rights Reserved. 54/90
(120). A 14-year-old girl with episodes of palpitations, chest pain, shortness of breath, and diaphoresis who has a normal physical and
laboratory examination

a. OCD

b. Hypochondriasis

c. Tourette disorder

d. Panic disorder

Solution. (d) Panic disorder


Ref:Read the text below
Sol:
- Palpitations, chest pain, shortness of breath, and diaphoresis commonly occur during panic attacks.
- These symptoms do not occur routinely in any of the other disorders.

Correct Answer. d

(121). Which of the following factors is strong indication of electroconvulsive therapy (ECT)?

a. Severe, melancholic depression and history of poor response to medications

b. Severe, adverse reactions to all selective serotonin reuptake inhibitors (SSRIs), tricyclic antidepressants (TCAs), and continued
suicidal ideation

c. Known history of psychotic symptoms in the context of her depressive symptoms

d. Known history of bipolar illness

Solution. (a) Severe, melancholic depression and history of poor response to medications
Ref:Read the text below
Sol:
- Depression severity, history of poor response to many medications (many clinicians believe that all medication options should be
exhausted), and the need for quick antidepressant action are really the only universally accepted criteria that encourage the clinician to
use ECT in major depression.
- Poor compliance would push a clinician toward ECT only if all other methods of encouraging compliance had failed.
- The other options all describe diagnoses that would likely respond to ECT but would not necessarily move the clinician to ECT over
medications.

Correct Answer. a

(122). A 62-year-old woman presents with a history of bilateral temporal lobectomy for an intractable seizure disorder.She is extremely docile
and displays very little emotion. She has a large appetite and compulsively puts both food and nonfood items in her mouth. She also
displays sexual disinhibition, often walking out of her room without her pants on. This patient’s clinical condition is most consistent with
which of the following?

a. Pick disease

b. Klüver-Bucy syndrome

c. Arnold-Chiari syndrome

d. punchdrunk syndrome

Solution. (b) Klüver-Bucy syndrome


Ref:Read the text below
Sol:
- Klüver-Bucy syndrome presents with docility, lack of fear response, anterograde amnesia, hyperphagia, and hypersexuality.
- Pick disease is a form of dementia, often indistinguishable from Alzheimer disease, in which the frontal and temporal lobes are
atrophied.
- Arnold-Chiari syndrome describes a condition with hydrocephalus and cerebellar anatomic and functional abnormalities.
- Punchdrunk syndrome describes an acquired movement disorder associated with traumatic damage to the substantia nigra. Möbius
syndrome is congenital absence of the facial nerves and nuclei with resulting bilateral facial paralysis.

Correct Answer. b

Copyright © 2014 Delhi Academy of Medical Sciences, All Rights Reserved. 55/90
(123). A 21-year-old man is noted to be restless and constantly moving. He states that he feels as if he has to be moving all the time and is
uncomfortable if he sits still. Which of the following is this sensation known as?

a. Akathisia

b. Akinesia

c. Tardive dyskinesia

d. Rabbit syndrome

Solution. (a) Akathisia


Ref:Read the text below
Sol:
- All of the choices are possible side effects, either short term or long term, of neuroleptics. Akathisia is the subjective sensation of motor
and mental restlessness.
- Akinesia is a dysfunction of slowed or absent movement that can be associated with pseudoparkinsonism in the setting of neuroleptics.
- A dystonia, like akathisia and akinesia, can occur acutely and involve muscle rigidity and spasticity.
- Tardive dyskinesia is a late-onset neurologic effect of neuroleptics and can include tongue and lip movements and choreic movement of
the trunk or limbs.
- Rabbit syndrome is another late-onset side effect that involves fine, rhythmic movements of the lips.

Correct Answer. a

(124). Darier’s Sign is seen in

a. Urticaria pigmentosa

b. Papular ulticaria

c. Solar urticaria

d. Darier’s disease

Solution. (a) Urticaria pigmentosa


Ref: Harrison-333.
Sol :
- Urticaria pigmentosa : A generalized distribution of red-brown macules and papules; also known as mastocytosis. Each lesion
represents a collection of mast cells in the dermis, with hyperpigmentation of the overlying epidermis.
- Darier’s sign; stimuli such as rubbing cause these mast cells to degranulate, and this leads to the formation of localized urticaria.
- Additional symptoms can result from mast cell degranulation and include headache, flushing, diarrhea, and pruritus.

Correct Answer. a

(125). Griseofulvin is not useful in

a. Tinea capitis

b. Tinea cruris

c. Tinea versicolor

d. Tinea pedis

Solution. (c) Tinea versicolor


Ref: KD Tripathi- 760-761
Sol :
- Griseofulvin is active against most dermatophyte (tinea) infections – Epidermophyton, Trichophyton and Microsporum, but not against
Candida and deep mycoses.
- It is indicated for the treatment of following tinea (dermatophyte/ringworm) infections; tinea corporis, tinea pedis (athlete’s foot), tinea
cruris, tinea barbae (barber’s itch), tinea capitis, and tinea unguium (onychomycosis)

Correct Answer. c

Copyright © 2014 Delhi Academy of Medical Sciences, All Rights Reserved. 56/90
(126). Ecthyma Gangrenosum is caused by

a. Streptococcus

b. Staphylococcus

c. Pseudomonas

d. Clostridium

Solution. (c) Pseudomonas


Ref: Harrison-335.
Sol :
- Ecthyma gangrenosum (EG) is classically associated with is Pseudomonas aeruginosa. EG usually occurs in patients who are critically ill
and immunocompromised and is almost always a sign of pseudomonal sepsis.
- Lesions of EG begin as edematous, erythematous papules or plaques and then develop central purpura and necrosis.
- Bullae formation also occurs in these lesions, and they are frequently found in the girdle region.

Correct Answer. c

(127). Scl-70 antibody is characteristic of

a. Systemic Lupus Erythematosus

b. Scleroderma

c. Dermatomyositis

d. Sjogren’s Syndrome

Solution. (b) Scleroderma


Ref: Harrison-2097
Sol :
- Characteristic autoantibodies are Anticentromere in limited scleroderma/CREST syndrome and Antitopoisomerase I (Scl-70) in diffuse
scleroderma.

Correct Answer. b

(128). A21-year-old woman diagnosed with panic disorder presents with increased frequency of panic attacks and complains of feeling as if her
surrounding environment is unreal and strange. As a result of the increasing frequency of her attacks and this new symptom, she has
been unwilling to leave her apartment for several weeks. How is the feeling of a surreal environment best described?

a. Depersonalization

b. Derealization

c. Hypermnesia

d. Dereism

Solution. (b) Derealization


Ref:Read the text below
Sol:
- Both depersonalization and derealization can be seen in anxiety disorders.
- Derealization is the sense that one’s surroundings are strange or unreal, and depersonalization is the feeling that one’s identity is lost
or the feeling of being unreal or strange.
- Dereism is simply mental activity not in accordance with reality.
- Hypermnesia is an abnormal recall of details. Paresthesia is an abnormal sensation such as tingling or prickling.

Correct Answer. b

Copyright © 2014 Delhi Academy of Medical Sciences, All Rights Reserved. 57/90
(129). 24-year-old man presents with new-onset command suicidal auditory hallucinations. He is given haloperidol 10 mg IM. Fifteen hours
later, he develops torticollis. What is the best treatment for this occurrence?

a. Acetaminophen

b. Labetalol

c. Benztropine

d. A lower-potency neuroleptic

Solution. (c) Benztropine


Ref:Read the text below
Sol:
- Benztropine 1–2 mg IM is useful in the treatment of acute dystonic reactions. Alternatively, diphenhydramine 50 mg IM or IV can be
used.
- If the symptoms do not resolve within 20 minutes, larger doses can be given. Benzodiazepines can also be tried but are not first-line
treatment.
- For acute laryngeal dystonia, 4 mg of benztropine should be given within 10 minutes, then 2 mg of lorazepam IV if needed.
- None of the other choices are indicated for the treatment of acute dystonia, although switching to a lowerpotency neuroleptic might be
considered as prophylaxis against further dystonic reactions.
- Labetalol is an antihypertensive, acetaminophen is an analgesic and antipyretic, and penicillamine is a chelating agent used to treat
Wilson disease.

Correct Answer. c

(130). A28-year-old woman with a history of bipolar disorder is admitted to the medical service because of weakness, mental status changes,
and a serum sodium of 154 mmol/L. Over the last 2 weeks, she has had polyuria, excessive thirst, and polydipsia. What drug is most likely
associated with this medical condition?

a. Lithium carbonate

b. Haloperidol

c. Diazepam

d. Valproic acid

Solution. (a) Lithium carbonate


Ref:Read the text below
Sol:
- The disorder described is diabetes insipidus.
- Lithium inhibits the effect of antidiuretic hormone on the kidney.
- Although haloperidol (an antipsychotic), diazepam (a benzodiazepine), valproic acid (an antiepileptic used as a mood stabilizer), and
buspirone (an anxiolytic/antidepressant) may be used to treat various manifestations of bipolar disorder, none of them are associated
with diabetes insipidus.

Correct Answer. a

(131). A 26-year-old man presents with paranoia, visual hallucinations, feelings of unreality, depersonalization, and extreme agitation. A urine
toxicology screen is positivefor phencyclidine (PCP). What is the best treatment for extreme agitation in this patient?

a. A phenothiazine antipsychotic

b. A butyrophenone antipsychotic

c. Trihexphenidyl

d. Trazodone

Solution. (b) A butyrophenone antipsychotic


Ref:Read the text below
Sol:
- For extreme agitation, the butyrophenone antipsychotic haloperidol is useful. Benzodiazepines are also useful for this symptom.
- Phenothiazine antipsychotics can cause autonomic instability when given to a patient with PCP intoxication.
- Trihexyphenidyl is used to combat extrapyramidal symptoms associated with antipsychotic use.
- Trazodone, used commonly for insomnia, and fluoxetine, an SSRI, are antidepressants.

Correct Answer. b

Copyright © 2014 Delhi Academy of Medical Sciences, All Rights Reserved. 58/90
(132). All the following statements regarding bulimia are true except

a. Binge eating is usually followed by a sense of euphoric well-being

b. Bulimic episodes may occur in persons suffering from anorexia nervosa

c. Bulimic persons are well aware of their disordered eating patterns

d. It most often affects young women

Solution. (a) Binge eating is usually followed by a sense of euphoric well-being Ref:Read the text below
Sol:
- Bulimia is an eating disorder in which copious amounts of food are ingested rapidly over a brief time period (usually less than 2 hours).
- Affected persons are painfully aware that such binge eating is unhealthy but feel that the impulsive eating is beyond control.
- Binges are characteristically followed by a profound sense of failure and depression.This condition tends to persist for several years,
with occasional remissions.
- It mostcommonly affects adolescent or young adult women.
- First symptoms may be associated with a major life change, such as leaving home or starting work. Bulimia may also occur as one
aspect of the related eating disorder anorexia nervosa.

Correct Answer. a

(133). Which of the following statements concerning the kinetics and clinical effects of benzodiazepines is true?

a. Short-half-life benzodiazepines are associated with a later onset of withdrawal signs than long-half-life benzodiazepines

b. Long-half-life benzodiazepines may be associated with residual sedation even weeks after the medication is discontinued

c. The metabolism of short-half-life agents is more influenced by old age than is that of long-half-life agents

d. Long-half-life agents are more efficacious in the treatment of anxiety symptoms

Solution. (b) Long-half-life benzodiazepines may be associated with residual sedation even weeks after the medication is discontinued
Ref:Read the text below
Sol:
- One way to begin to differentiate clinically among the many benzodiazepines currently marketed for the treatment of anxiety is to divide
them according to half-life. The long-acting agents include diazepam and chlordiazepoxide. They are metabolized in several steps, by
hepatic oxidation, and therefore have biologically active metabolites. The short-acting agents include oxazepam and lorazepam.
- They are metabolized by glucuronide conjugation to inactive metabolites. Both groups are equally efficacious in the treatment of
anxiety, in either single-dose or multiple-dose regimens. However, when multiple-dose treatment is indicated, it is important to be aware
of the pharmacokinetic difference between these agents. The time lag before onset of medication effect will depend on the absorption
rate.
- Diazepam has been found to act more rapidly and more profoundly than the less well absorbed oxazepam. However, because of longer
half-life, diazepam accumulates extensively in the body during multiple-dose regimens, with corresponding clinical side effects such as
excessive sedation.
- Because of their slow elimination rate, long-acting agents may produce sedation 2 weeks or more after the agent is discontinued. There
may also be increased risk of dangerous interaction with other substances, including alcohol.
- On the other hand, short-acting agents accumulate less and are more rapidly eliminated. Abrupt discontinuation of these agents may
precipitate sudden onset of withdrawal symptoms, including increased anxiety, insomnia, and autonomic nervous system disturbance.
- One distinction between these agents that must be emphasized is that hepatic oxidation tends to be much more influenced by the aging
process than does the glucuronidation process by which the short-acting agents are metabolized. Thus, elderly individuals may be much
more prone to the dangers of drug accumulation when receiving long-half-life benzodiazepines. This is the basis for the opinion that
short-half-life benzodiazepines are the drugs of choice in treating anxiety in the geriatric population.

Correct Answer. b

Copyright © 2014 Delhi Academy of Medical Sciences, All Rights Reserved. 59/90
(134). A severely emotionally ill woman, diagnosed as having schizoaffective illness, takes the following psychotropic medications: nortriptyline,
fluphenazine, benztropine, lorazepam, and triazolam. She consults her gynecologist because of a variety of complaints, and
hyperprolactinemia is discovered. Of the woman’s medications, the one most likely to be causing her hyperprolactinemia is

a. Nortriptyline

b. Fluphenazine

c. Benztropine

d. Lorazepam

Solution. (b) Fluphenazine


Ref:Read the text below
Sol:
- Among psychotropic drugs, antipsychotic preparations are most likely to cause hyperprolactinemia.
- Release of prolactin is inhibited by dopamine.
- Therefore, the central dopamine receptor blockade produced by antipsychotic drugs leads to elevated prolactin production.

Correct Answer. b

(135). Antisocial personality disorder is considered to be

a. Synonymous with criminal behavior

b. The male counterpart of histrionic personality disorder

c. Amenable to pharmacologic treatment with antiandrogenic agents

d. Correlated with electroencephalographic studies suggesting cortical immaturity

Solution. (d) Correlated with electroencephalographic studies suggesting cortical immaturity


Ref:Read the text below
Sol:
- Antisocial personality disorder represents a chronically maladaptive pattern of interaction and perception in which the rights of others
are violated and in which socialproductivity (sustained job performance or intimate relationships) is impaired.
- Although an individual with antisocial personality disorder might engage in criminal acts, criminality as a socially defined behavior is
determined by a variety of economic, cultural, and other environmental factors.
- Family studies have shown that antisocial personality is associated with several other psychiatric disorders, including histrionic
personality disorder and alcoholism, which are diagnosable in first-degree relatives at higher rates than among the general population.
These associations raise the possibility that the disorders may have common genetic as well as environmental determinants.
- Electroencephalographic studies reveal an increased prevalence of slow-wave activity, lowered thresholds for sedation, and slow
cortical recovery from stimulation, suggesting that individuals with antisocial personality disorder manifest signs of cortical immaturity.
Pharmacologic intervention has no proven efficacy in the treatment of antisocial personality.
- Early psychologic and behavioral interventions seem to improve long-term outcome

Correct Answer. d

(136). The so-called first-rank, or schneiderian, symptoms of schizophrenia include all the following except

a. Auditory hallucinations

b. Thought broadcasting

c. Thought insertion

d. Catatonia

Solution. (d) Catatonia


Ref:Read the text below
Sol:
- Symptoms of schizophrenia have been classified in a number of ways, beginning with Eugen
- Bleuler’s “four A’s”: autism, ambivalence, associational difficulties, and affect disturbance. Kurt Schneider 30 years ago described what
he termed “first rank” symptoms, which, if present and not associated with a known organic disorder, would indicate a diagnosis of
schizophrenia.
- Among Schneider’s first-rank symptoms are auditory hallucinations, somatic hallucinations, delusions, and the belief that thoughts
either are being intruded on by others’ thoughts or are being telegraphed to others and incorporated into their thoughts (thought
insertion and thought broadcasting, respectively).
- Catatonia is not a first-rank symptom of schizophrenia.

Correct Answer. d

Copyright © 2014 Delhi Academy of Medical Sciences, All Rights Reserved. 60/90
(137). Electroconvulsive therapy (ECT) can be described by which of the following statements?

a. ECT is contraindicated in the treatment of psychotic depression

b. ECT can cause depressed persons to exhibit manic symptoms

c. Transient memory loss occurs rarely after each ECT treatment

d. Morbidity and mortality are greater than with antidepressant drug therapy

Solution. (b) ECT can cause depressed persons to exhibit manic symptoms Ref:Read the text below
Sol:
- Although the advent of antidepressant drugs has curtailed the use of electroconvulsive therapy (ECT), the procedure still is employed to
treat persons with psychotic depression, medication-resistant depression, and florid mania.
- Protocols vary among treatment centers in regard to frequency and number of shocks, strength of shock, and method of administration
(unilateral or bilateral).
- The use of pretreatment medication, including atropine to dry up secretions and succinylcholine to cause transient paralysis, has
lowered the morbidity and mortality of ECT to levels less than those associated with antidepressant drug therapy. Transient memory loss
usually occurs after each treatment.

Correct Answer. b

Copyright © 2014 Delhi Academy of Medical Sciences, All Rights Reserved. 61/90
(138). Paranoid psychosis observed with cocaine abuse can be explained by –

Copyright © 2014 Delhi Academy of Medical Sciences, All Rights Reserved. 62/90
a. Tolerance

b. Intoxication

c. Reverse tolerance

d. Withdrawal

Solution. (c) Reverse tolerance


Ref:Read the text below
Sol :
Cocaine is a stimulant and a localanesthetic with potent vasoconstrictor properties. The reinforcing effects of cocaine appear to be
related to activation of dopaminergic neurons in the mesolimbic system. Cocaine increases synaptic concentrations of the
monamine neurotransmitters dopamine, norepinephrine, and serotonin by binding to transporter proteins in presynaptic
neurons and blocking reuptake.
Forms of the drug
In powder form, cocaine is known by such street names as "coke," "blow," "C," "flake," "snow" and "toot." It is most commonly inhaled or
"snorted." It may also be dissolved in water and injected.
Crack is a smokable form of cocaine that producesan immediate and more intense high. It comes in off-white chunks or chips called
"rocks." Little crumbs of crack are sometimes called "kibbles & bits."
In addition to their stand-alone use, both cocaine and crack are often mixed with other substances. Cocaine may be mixed with
methcathinone (a more recent drug of abuse, known as "cat," that is similar to methamphetamine) to create a "wildcat." A
hollowed-out cigar filled with a mixture of crack and marijuana is known as a "woolah." And either cocaine or crack used in conjunction
with heroin is called a "speed-ball." Cocaine used together with alcohol represents the most common fatal two-drug combination.
Cocaine produces a brief, dose-related stimulation and enhancement of mood and an increase in cardiac rate and blood pressure. Body
temperature usually increases following cocaine administration, and high doses of cocaine may induce lethal pyrexia or hypertension.
Because cocaine inhibits reuptake of catecholamines at adrenergic nerve endings, the drug potentiates sympathetic nervous system
activity. Cocaine has a short plasma half-life of approximately 45–60 min. It is metabolized by plasma esterases, and cocaine metabolites
are excreted in urine. The very short duration of the euphorigenic effects of cocaine observed in chronic abusers is probably
due to both acute and chronic tolerance.
Cocaine abuse has four distinct phases-
· Euphoria
· Dysphoria
· Halucination
· Psychosis

1. The first phase is cocaine euphoria, characterized by intense pleasure which is accompanied by affective lability, hypervigilance,
hyperactivity and hypersexuality.

2. The second phase, which sometimes follows after a few hours of smoking, is a dysphoric state manifested by
considerable anxiety and smoking compulsion. Other affective changes can be observed in some persons as sadness, melancholy, apathy,
or aggressiveness..

3. The third phase is the cocaine hallucinosis, which can also develop after smoking several grams of coca paste and is marked by visual,
tactile, auditory, and olfactory hallucinations. The patient is generally very excited and has transitory delusional interpretations. The
hallucinosis is evanescent, it may last for two or three days and then fade gradually, if the individual has discontinued to smoke or
receives parenteral neuroleptics.

4. The fourth phase is the cocaine psychosis. It may appear after days or weeks of frequent or continued paste smoking. It is
characterized by a marked agitation with hypervigilance and defined paranoid delusions of persecution, damage, death or spouse
unfaithfulness. Generally it is accompanied by auditory and olfactory hallucinations.

Treatment: Cocaine Overdose


Treatment of cocaine overdose is a medical emergency that is best managed in an intensive care unit. Cocaine toxicity produces a
hyperadrenergic state characterized by hypertension, tachycardia, tonic-clonic seizures, dyspnea, and ventricular arrhythmias.
Intravenous diazepam in doses up to 0.5 mg/kg administered over an 8-h period has been shown to be effective for control
of seizures. Ventricular arrhythmias have been managed successfully by administration of 0.5–1 mg of propranolol IV. Since
many instances of cocaine-related mortality have been associated with concurrent use of other illicit drugs (particularly heroin).
Cocaine withdrawl
If a person who is dependent on cocaine suddenly stops taking it, or significantly cuts down the amount they are using, they can
experience withdrawal symptoms.
Cocaine withdrawal generally occurs in three phases: the ‘crash’; the ‘withdrawal’; and the ‘extinction’.
· The crash, which usually occurs in the first few days, describes the withdrawal symptoms experienced immediately after the person
stops using cocaine.
· The withdrawal phase may last up to 10 weeks. During this phase, people experience severe cravings for cocaine.
· The extinction phase may last indefinitely. It involves intermittent cravings for cocaine. These generally occur in response to people,
places or objects that are conditioned cues and provoke memories of taking the drug. These cravings may surface months or years after
cocaine use has stopped.
Reverse Tolerance-
As the tolerance to cocaine increases, the addict feels he is becoming less sensitive to the drug’s effects. While the addict is
becoming less sensitive to the pleasurable effect of the drug, he is at the same time becoming more likely to develop
epileptic seizures, delusions, and psychotic behavior.

Correct Answer. c

Copyright © 2014 Delhi Academy of Medical Sciences, All Rights Reserved. 63/90
(139). A 35-year female has been diagnosed with obsessive compulsive disorder (OCD) and she washes her hands 40 times a day. Which would
be the best CBT technique for her treatment?

a. Thought stopping

b. Response prevention

c. Relaxation

d. Exposure

Solution. (b) Response prevention


Ref: Read the text below
Sol :
Obsessive-Compulsive Disorder Obsessive-compulsive disorder (OCD) is characterized by obsessive thoughts and compulsive behaviors
that impair everyday functioning. Fears of contamination and germs are common, as are handwashing, counting behaviors, and having to
check and recheck such actions as whether a door is locked. The degree to which the disorder is disruptive for the individual varies, but
in all cases obsessive-compulsive activities take up >1 hour per day and are undertaken to relieve the anxiety triggered by the core fear.
Patients often conceal their symptoms, usually because they are embarrassed by the content of their thoughts or the nature of their
actions. A genomewide association study (GWAS) reported linkage to chromosome 2p23.2; however, no susceptibility gene for OCD has
been identified to date. Family studies show an aggregation of OCD with Tourette's disorder, and both are more common in males and in
first-born children. Treatment: Obsessive-Compulsive Disorder
Clomipramine, fluoxetine, fluvoxamine, and sertraline are approved for the treatment of OCD. Clomipramine is a TCA that is often
tolerated poorly owing to anticholinergic and sedative side effects at the doses required to treat the illness (25–250 mg/d); its efficacy in
OCD is unrelated to its antidepressant activity. Fluoxetine (5–60 mg/d), fluvoxamine (25–300 mg/d), and sertraline (50–150 mg/d) are as
effective as clomipramine and have a more benign side effect profile. Only 50–60% of patients with OCD show adequate improvement
with pharmacotherapy alone. In treatment-resistant cases, augmentation with other serotonergic agents such as buspirone, or with a
neuroleptic or benzodiazepine may be beneficial and in severe cases deep brain stimulation has been found to be effective. When a
therapeutic response is achieved, long-duration maintenance therapy is usually indicated.
For many individuals, particularly those with time-consuming compulsions, behavior therapy will result in as much improvement as that
afforded by medication. Effective techniques include the gradual increase in exposure to stressful situations, maintenance of a diary to
clarify stressors, and homework assignments that substitute new activities for compulsive behaviors.

Correct Answer. b

(140). Poor scholastic performance is not associated with –

a. SLD

b. ADHD

c. PICA

d. Anxiety

Solution. (c) PICA


Ref: Read the text below
Sol :
In PICA presenting complaints is either nutritional deficiencies, lead complications, or persistantly taking something in mouth.
ADHD,Anxiety & specific learning disorder are associated with decreased scholastic performance.
Pica is characterized by an appetite for substances largely non-nutritive (such as clay or chalk). For these actions to be considered pica,
they must persist for more than one month at an age where eating such objects is considered developmentally inappropriate. There are
different variations of pica, as it can be from a cultural tradition, acquired taste or a neurological mechanism such as an iron deficiency,
or chemical imbalance. Diagnosis
There is no single test that confirms pica. However, because pica can occur in people who have lower than normal nutrient levels and
poor nutrition (malnutrition), the health care provider should test blood levels of iron and zinc. Hemoglobin can also be checked to test
for anemia. Lead levels should always be checked in children who may have eaten paint or objects covered in lead-paint dust. The health
care provider should test for infection if the person has been eating contaminated soil or animal waste. DSM IV criteria for PICA
A. Persistent eating of nonnutritive substances for a period of at least 1 month
B. Does not meet the criteria for either autistic disorder, schizophrenia, or Kleine-Levin syndrome.
C. The eating behavior is not part of a culturally sanctioned practice
D. If the eating behavior occurs exclusively during the course of another mental disorder (e.g., mental retardation, pervasive
developmental disorder, schizophrenia), it is sufficiently severe to warrant independent clinical attention.

Correct Answer. c

Copyright © 2014 Delhi Academy of Medical Sciences, All Rights Reserved. 64/90
(141). A 16-year-old girl presents with complaints of abdominal pain and vomiting. The girl is emotionally high strung and has irregular patchy
loss of hair. The next step in diagnosing this case is?

a. KOH scraping

b. Skin biopsy

c. Surgical referral

d. Psychiatric evaluation

Solution. (d) Psychiatric evaluation


Ref:Read the text below
Sol:
Trichotillomania may be associated with eating up of the plucked hair (trichophagia). For the diagnosis of this patient referral to
psychiatrist is recommended. For management surgical intervention may be required.
- Due to self-induced pulling/twisting of hairs in children/adolescents with psychological problems present with alopecia without
inflammation.
- The margin of the lesion is less well defined and there are no exclamation mark hairs.
- Short broken hairs of varying length are characteristic.
- Tonsure alopecia or Orentriech sign with maximum loss from fronto-parietal areas & sparing of peripheral hairs: The hair loss is never
complete. from accessible areas leading to “friar tuck appearance”.
- Refer to clinical psychologist or psychiatrist.

Correct Answer. d

(142). 18-year-old male has nail pitting and patchy hair loss. What else is expected?

a. Auspitz sign

b. Oreintreich sign

c. Frair-tuck appearance

d. Exclamation mark hair

Solution. (d) Exclamation mark hair


Ref:Read the text below
Sol:
- Pitting of nail is seen in Psoriasis, Alopecia areata, Nail trauma or biting and chronic eczemas. This patient also has patchy loss of hair.
- Most likely this is a case of Alopecia areata and hence Exclamation mark hair are expected to be seen.

Correct Answer. d

(143). A patient comes with transverse grooves in multiple nails. What is likely?

a. It demonstrates a tick deformity with habitual nail biting

b. Possibility of onychomycosis requiring a KOH study

c. A possible severe illness in past few months

d. Transverse pterygium indicating lichen planus

Solution. (c) A possible severe illness in past few months


Ref:Read the text below
Sol:
- Beau's lines: Deep transverse groove caused by interference with nail formation during a period of stress caused by malnutrition,
infection, illness and trauma and hence are at almost the same level in all nails.

Correct Answer. c

Copyright © 2014 Delhi Academy of Medical Sciences, All Rights Reserved. 65/90
(144). A patient presents with atrophy of skin, reticulate pigmentation and minute telengiectasias. The diagnosis is

a. Scleroderma

b. Atrophoderma

c. Dyskeratosis

d. Poikiloderma

Solution. (d) Poikiloderma


Ref:Read the text below
Sol:
- Poikiloderma is defined as a triad of Atrophy, Reticulate pigmentation and telengiectasias.

Correct Answer. d

(145). Volcano sign is seen in

a. Pyogenic granuloma

b. Pyoderma gangrenosum

c. Leishmaniasis

d. Pityriasis versicolor

Solution. (c) Leishmaniasis


Ref:Read the text below
Sol:
- Volcano sign is the presence of a lesion with central crater like depression and is characterstic of cutaneous leishmaniasis.

Correct Answer. c

Copyright © 2014 Delhi Academy of Medical Sciences, All Rights Reserved. 66/90
(146). Identify the procedure in Fig 6 :

a. Repetitive Transcranial Magnetic Stimulation

b. Vagal Nerve Stimulation

c. Deep Brain Stimulation

d. Electro Convulsive Therapy

Solution. (a) Repetitive Transcranial Magnetic Stimulation


Ref: Read the text below
Sol:
- Repeated transcranial magnetic stimulation (rTMS) is a noninvasive technique for stimulating cells of the cerebral cortex.
- It creates a time-varying magnetic field in which a localized pulse magnetic field over the surface of the head depolarizes the superficial
neurons.
- TMS uses a hand-held magnet to allow focused electrical stimulation across the scalp and cranium without the pain associated with
percutaneous electrical stimulation.
- If TMS pulses are delivered repetitively and rhythmically, the technique is called rTMS. rTMS was originally used to map cortical motor
control and hemisphere dominance.

Correct Answer. a

(147). What is not seen in lichen planus

a. Excoriation marks

b. Purpulish papules

c. Lacy white lines in buccal mucosa

d. Koebenerization

Solution. (a) Excoriation marks


Ref:Read the text below
Sol:
- Lichen planus is characterized by pruritic, purpulish, polygonal, polished and plane-topped papules with Wikham's striae on the skin,
mainly at the flexural aspect of wrist. Mucosal lesions are reticular white lines while nails are characterized by dorsal pterygium and sclp
involvement produces cicatricial alopecia.
- Koebenerization or isomorphic response is typical of lichen planus where lesions appear at sites of trauma.
- However, the pruritus of lichen planus is a desire to rub and not to scratch.
- Hence excoriation marks are typically not sen in lichen planus.

Correct Answer. a

Copyright © 2014 Delhi Academy of Medical Sciences, All Rights Reserved. 67/90
(148). A 53 yr old lady on immunoblotting shows Ig G against desmoglein 3. She is likely to present with?

a. Oral erosions

b. Tense bullae

c. Grouped itchy excoriated papules on elbows and knees

d. Periorificial erosions

Solution. (a) Oral erosions


Ref:Read the text below
Sol:
- IgG against desmoglein 3 is seen in pemphigus vulgaris.
- Almost 90-95% of patients have mucosal involvement while nearly 70% of them present with oral erosions.
- Flaccid blisters on skin usually develop late or simultaneously with oral lesions.

Correct Answer. a

(149). The following drug is indicated in the treatment of P versicolor

a. Ketoconazole

b. Metronidazole

c. Griesofulvin

d. Chloroquine

Solution. (a) Ketoconazole


Ref:Read the text below
Sol:
- Pityriasis versicolor is not responsive to griesofulvin.
- It can be treated by other antifungals like fluconazole, ketoconazole, itraconazole systemically and miconazole,clotrimazole topically.
- It can also be treated with topical Whitfield's ointment, selenium sulfide etc.

Correct Answer. a

(150). A 24 yr old male presents to a STD clinic with a single painless ulcer on external genetitalia. The choice of laboratory test to look for the
etiological agent would be

a. Srapings from ulcer for culture on chocolate agar with antibiotic suuplement

b. Serology for detection of specific IgM antibodies

c. Scrapings from the ulcer for dark field microscopy

d. Scrapings from ulcer for tissue culture

Solution. (c) Scrapings from the ulcer for dark field microscopy
Ref:Read the text below
Sol:
- As a thumb rule all cases of genital ulcers should be examined with a dark ground microscopy.
- In this case since it is likely a case of syphilis (single,painless ulcer) it is even more so the best option.

Correct Answer. c

Copyright © 2014 Delhi Academy of Medical Sciences, All Rights Reserved. 68/90
(151). Podophyllin resin is indicated in the treatment of

a. Psoriasis

b. Pemphigus

c. Condyloma acuminata

d. Condyloma lata

Solution. (c) Condyloma acuminata


Ref:Read the text below
Sol:
Podophyllin resin in the tincture of benzoin is the recommended treatment for condyloma acuminata (HPV infection). Condyloma lata is
secondary syphilitic lesion and is treated with benzathine penicillin injection. Psoriasis and pepmphigus require individualized
treatments. Treatment options for condyloma acuminata: Physician administered: Podophyllin resin, Trichloroacetic acid
application,Cryotherapy, electrosurgery and laser surgery Patient applied: Podofilox cream, Imiquimod

Correct Answer. c

(152). A 45 yr old male has multiple grouped vesicular lesions present on the T-10 dermatome associated with pain. Most likely diagnosis is

a. Herpes zoster

b. Dermatitis herpetiformis

c. Herpes simplex

d. Scabies

Solution. (a) Herpes zoster


Ref:Read the text below
Sol:
- Herpes zoster clinically presents with unilateral, segmental or dermatomal grouped, vesicular response. It is preceded, accompanied
and followed by severe pain. The pain following the lesions is called the post herpetic neuralgia.
- Herpes simplex is more itchy than painful and is not dermatomal. Dermatitis herpetiformis is grouped, itchy papulovesicles on
extensors like knees, elbow,back etc.
- Erythema multiforme is usually asociated with herpes simplex infections and presents with target type lesions on the acral areas like
hands,

Correct Answer. a

Copyright © 2014 Delhi Academy of Medical Sciences, All Rights Reserved. 69/90
(153). A 28 year old male has multiple grouped papulovesicles on both elbows, knees, buttocks, upper back associate with severe itching. The
most likely diagnosis is

a. Pemphigus vulgaris

b. Dermatitis Herpetiformis

c. Bullous pemphigoid

d. Herpes zoster

Solution. (b) Dermatitis Herpetiformis


Ref:Read the text below
Sol:
This is classical clinical picture of Dermatitis herpetiformis. An uncommon skin condition with the peak incidence around 30-40 years of
age (M>F). Again, DH has nothing to do with Herpetic infection. Pathogenesis: disease usually associates with celiac disease (Gluten
sensitive enteropathy). And many other antibodies are found for eg. Anti-endomysial,anti transglutaminase (? Pathogenic) etc. and they
deposit at or around BMZ leading to defects at these sites with resulting clefting. Clinical Features
- Intense itching
- Grouped paulovesicles which are usually excoriated and scabbed and needs to be differentiated from Scabies and acute atopic
dermatitis.
- The sites of predilection are elbows and knees, buttocks, upper back and face.
- Intact vesicles seldom seen since they are ruptured by self scratching.
- Lesions heal with no scarring.
Investigations
- Histology: Neutrophilic Microabscess in dermal papillary tip is characteristic.
- DIF: It shows granular IgA deposition in BMZ.
- Screening for the presence of celiac disease.
- Provocation test: iodides in diet or patch.
Treatment
- DAPSONE: It is the DOC. After starting the treatment, symptoms should subside within 48 hours and rash should be cleared up within
days.
- Sulfapyridine/sulphamethoxypyridazine: when dapsone is contraindicated.
- Gluten Free diet: It is useful but Dietary restriction is not easy to follow.

Correct Answer. b

(154). What is not a standard treatment option for extensive psoriasis vulgaris?

a. Methotrexate

b. Cyclosporine

c. Acitretin

d. Betatmethasone

Solution. (d) Betatmethasone


Ref:Read the text below
Sol:
- Systemic corticosteroids should not be used for treatment of psoriasis as it leads to relapses on withdrawl and sudden withdrawl by the
physician or the patient may lead to a life threatning pustular flare.
- Methotrexate is wdely used for extensive psoriasis and cyclosporine is also very effective. Systemic retinoids are very useful in psoriasis
eg Acitretin, espin pustular psoriasis.
- The only true indication of systemic corticosteroids in psoriasis is the life threatening putular psoriasis in pregnancy ( Impetigo
Herprtiformis) where other drug classes are contraindicated and steroids are used to tide over the crisis.

Correct Answer. d

Copyright © 2014 Delhi Academy of Medical Sciences, All Rights Reserved. 70/90
(155). Delusions are not likely to be a part of the clinical picture of –

a. Dementia

b. Schizophrenia

c. Depression

d. Conversion disorder

Solution. (d) Conversion disorder


Ref: Read the text below
Sol :
The definition of a delusion is a false,unshakeable idea or belief held with extraordinary conviction and subjective certainty.Delusion are
common symptom of arrange of psychiatric illnesss including Schizophrenia,depression,bipolar disorder & dementia. DSM IV criteria for
Conversion disorder-
A. One or more symptoms or deficits affecting voluntary motor or sensory function that suggest a neurological or other general medical
condition. B. Psychological factors are judged to be associated with the symptom or deficit because the initiation or exacerbation of the
symptom or deficit is preceded by conflicts or other stressors.
C. The symptom or deficit is not intentionally produced or feigned (as infactitious Disorder or Malingering).
D. The symptom or deficit cannot, after appropriate investigation, be fully explained by a general medical condition, or by the direct
effects of substance, or as a culturally sanctioned behavior or experience. E. The symptom or deficit causes clinically significant distress
or impairment in social, occupational, or other important areas of functioning or warrants medical evaluation.
F. The symptom or deficit is not limited to pain or sexual dysfunction, does not occur exclusively during the course of Somatization
Disorder, and is not better accounted for by another mental disorder. Specify type of symptom or deficit: With Motor Symptom or Deficit
With Sensory Symptom or Deficit With Seizures or Convulsions With Mixed Presentation

Correct Answer. d

(156). Repetitive transcranial magnetic brain stimulation (rTMS) is approved by US FDA for the treatment of –

a. Depressive disorder

b. Obsessive compulsive disorder

c. Acute psychosis

d. Resistant schizophrenia

Solution. (a) Depressive disorder


Ref: Read the text below
Sol :
Treatment of Depression- The most effective intervention for achieving remission and preventing relapse is medication, but combined
treatment, incorporating psychotherapy to help the patient cope with decreased self-esteem and demoralization, improves outcome.
Electroconvulsive therapy is at least as effective as medication, but its use is reserved for treatment-resistant cases and delusional
depressions. Transcranial magnetic stimulation (TMS) is approved for treatment-resistant depression and has been shown to have
efficacy in several controlled trials. Vagus nerve stimulation (VNS) has also recently been approved for treatment-resistant depression,
but its degree of efficacy is controversial. Deep brain stimulation is another treatment that is being used experimentally in treatment
resistant cases.
Transcranial magnetic stimulation (TMS) is a noninvasive method to cause depolarization or hyperpolarization in the neurons of the
brain. TMS uses electromagnetic induction to induce weak electric currents using a rapidly changing magnetic field; this can cause
activity in specific or general parts of the brain with minimal discomfort, allowing the functioning and interconnections
of the brain to be studied. A variant of TMS, repetitive transcranial magnetic stimulation (rTMS), has been tested as a treatment tool for
various neurological and psychiatric disorders including migraines, strokes, Parkinson's disease, dystonia, tinnitus, depression and
auditory hallucinations

Correct Answer. a

Copyright © 2014 Delhi Academy of Medical Sciences, All Rights Reserved. 71/90
(157). A 16-yr old boy is suffering from drug abuse and his perception of sensory modailities has crossed over, i.e. he complains that sounds can
be seen and sights can be heard. Which of the following agent is most likely responsible for this?

a. Cocaine

b. LSD

c. Marijuana

d. PCP

Solution. (b) LSD


Ref: Read the text below
Sol :
- LSD is a very potent hallucinogen; oral doses as low as 20 microgm may induce profound psychological and physiologic effects.
Tachycardia, hypertension, pupillary dilation, tremor, and hyperpyrexia occur within minutes following oral administration of 0.5–2
microgm/kg. A variety of bizarre and often conflicting perceptual and mood changes, including visual illusions, synesthesias, and extreme
lability of mood, usually occur within 30 min after LSD intake. These effects of LSD may persist for 12–18 h, even though the half-life of
the drug is only 3 h.
- The most frequent acute medical emergency associated with LSD use is a panic episode (the "bad trip"), which may persist up to 24 h.
Management of this problem is best accomplished by supportive reassurance ("talking down") and, if necessary, administration of small
doses of anxiolytic drugs. Adverse consequences of chronic LSD use include enhanced risk for schizophreniform psychosis and
derangements in memory function, problem solving, and abstract thinking. Treatment of these disorders is best carried out in specialized
psychiatric facilities.
- Tolerance develops rapidly for LSD-induced changes in psychological function when the drug is used one or more times per day for >4
days. Abrupt abstinence following continued use does not produce withdrawal signs or symptoms. There have been no clinical reports of
death caused by the direct effects of LSD.
- Phencyclidine (PCP)-PCP binds to ionotropic N-methyl-d-aspartate (NMDA) receptors in the nervous system, blocking ion current
through these channels. The most common street preparation, angel dust, is a white granular powder that contains 50–100% percent of
the drug. Low doses (5 mg) produce agitation, excitement, impaired motor coordination, dysarthria, and analgesia. Physical signs of
intoxication may include horizontal or vertical nystagmus, flushing, diaphoresis, and hyperacusis. Behavioral changes include distortions
of body image, disorganization of thinking, and feelings of estrangement. Higher doses of PCP (5–10 mg) may produce profuse salivation,
vomiting, myoclonus, fever, stupor, or coma. PCP doses of >10 mg cause convulsions, opisthotonus, and decerebrate posturing, which
may be followed by prolonged coma.

Correct Answer. b

(158). Which of the following statements regarding the NDPS act is true?

a. It provides for severe punishment for drug users and peddlars alike

b. Farmers growing opium are permitted to grow and consume unlimited amounts of opium

c. Consumption of excessive alcohol is punishable under the act

d. The act recommends treatment of drug users rather than sending them to jail

Solution. (a) It provides for severe punishment for drug users and peddlars alike Ref: Read the text below
Sol :
NDPS Act 1985- (Narcotic Drugs and Psychotropic Substances Act, 1985)
a) The cultivation, production, manufacture, possession, sale, purchase, transportation, warehousing, consumption, inter-State
movement, transshipment and import and export of narcotic drugs and psychotropic substances is prohibited, except for medical or
scientific purposes and in accordance with the terms and conditions of any license, permit or authorization given by the Government.
(Section 8)
b) The Central Government is empowered to regulate the cultivation production, manufacture, import, export, sale, consumption, use etc
of narcotic drugs and psychotropic substances. (Section 9). c) State Governments are empowered to permit and regulate possession and
inter-State movement of opium, poppy straw, the manufacture of medicinal opium and the cultivation of cannabis excluding hashish.
(Section 10). d) All persons in India are prohibited from engaging in or controlling any trade whereby narcotic drugs or psychotropic
substances are obtained outside India and supplied to any person outside India except with the previous authorisation of the Central
Government and subject to such conditions as may be imposed by the Central Government. (Section 12). e) The Central Government is
empowered to declare any substance, based on an assessment of its likely use in the manufacture of narcotics drugs and psychotropic
substances as a controlled substance. (Section 9-A). f) Assets derived from drugs trafficking are liable to forfeiture (Chapter V-A). g) Both
the Central Government and State Governments are empowered to appoint officers for the purposes of the Act.(Sections 4, 5 and 7). The
NDPS Act is in effect a comprehensive code not only for the control and regulation of Narcotics Drugs and Psychotropic Substances; but
also for the control of selected chemicals - commonly known as precursors - which can be used in the illicit manufacture of narcotic
drugs and psychotropic substances, as well as for the investigation and forfeiture of drug related assets.

Correct Answer. a

Copyright © 2014 Delhi Academy of Medical Sciences, All Rights Reserved. 72/90
(159). Bizarre interpretation of an object is most appropriately known as –

a. Illusion

b. Delusion

c. Hallucination

d. Dream

Solution. (a) Illusion


Ref: Read the text below
Sol :
Illusion- Perceptual misinterpretration of a real external stimulus. Hallucination- False sensory perception occurring in absence of any
relevant external stimulation of the sensory modality involved. Delusion- False belief , based on incorrect inference about external
reality,that is firmly held despite objective and obvious contradictory proof or evidence and despite the fact that other member of culture
do not share the belief. Dreamy state- Altered state of consciousness,likened to a dream situation,which develop suddenly and usually
lasts a few minutes,accompanied by visual,auditory,and olfactory hallucinations.

Correct Answer. a

(160). Paranoid schizophrenia can be described by which of the following statements?

a. Affected persons quickly lose the ability to function appropriately in social encounters

b. Age of onset generally is earlier than in other types of schizophrenia

c. Cognitive impairment tends to progress more rapidly than in other types of schizophrenia

d. Hallucinations and delusions often are grandiose in nature

Solution. (d) Hallucinations and delusions often are grandiose in nature


Ref:Read the text below
Sol:
- Paranoid schizophrenia is the most common subtype of schizophrenia.
- It tends to develop later in life than other schizophrenic subtypes, cause less cognitive impairment, and produce milder behavioral and
social disorganization.
- Hallucinations and delusions characteristically are of a jealous, persecutory, or grandiose nature.

Correct Answer. d

Copyright © 2014 Delhi Academy of Medical Sciences, All Rights Reserved. 73/90
(161). GAS (General Adaptation Syndrome) is seen in –

a. Panic attacks

b. Depression

c. Anxiety

d. Stressful situations

Solution. (d) Stressful situations


Ref: Read the text below
Sol :
Hans Selye developed a model of stress that he called the general adaptaion syndrome. General adaptive syndrome

A diagram of the General Adaptation Syndrome model. Physiologists define stress as how the body reacts to a stressor, real or imagined,
a stimulus that causes stress. Acute stressors affect an organism in the short term; chronic stressors over the longer term.
Alarm is the first stage. When the threat or stressor is identified or realized, the body's stress response is a state of alarm. During this
stage, adrenaline will be produced in order to bring about the fight-or-flight response. There is also some activation of the HPA axis,
producing cortisol Resistance is the second stage. If the stressor persists, it becomes necessary to attempt some means of coping with
the stress. Although the body begins to try to adapt to the strains or demands of the environment, the body cannot keep this up
indefinitely, so its resources are gradually depleted.
Exhaustion is the third and final stage in the GAS model. At this point, all of the body's resources are eventually depleted and the body is
unable to maintain normal function. The initial autonomic nervous system symptoms may reappear (sweating, raised heart rate, etc.). If
stage three is extended, long-term damage may result, as the body's immune system becomes exhausted, and bodily functions become
impaired, resulting in decompensation.
The result can manifest itself in obvious illnesses such as ulcers, depression, diabetes, trouble with the digestive system, or even
cardiovascular problems, along with other mental illnesses

Correct Answer. d

Copyright © 2014 Delhi Academy of Medical Sciences, All Rights Reserved. 74/90
(162). Which one of the following is not a feature of catatonic schizophrenia

a. Cataplexy

b. Catalepsy

c. Flexibilitas cereas

d. Command automatism

Solution. (a) Cataplexy


Ref:Read the text below
Sol:
- Cataplexy is a sudden and transient episode of loss of muscle tone, often triggered by emotions.
- It is a rare disease (prevalence of fewer than 5 per 10,000 in the community), but frequently affects people who have narcolepsy, a
disorder whose principal signs are EDS (Excessive Daytime Sleepiness), sleep attacks, sleep paralysis,hypnagogic hallucinations and
disturbed night-time sleep.
- Cataplexy is sometimes confused with epilepsy.It is seen in catatonic schizophrenia.

Correct Answer. a

(163). Wax moulding posturing is characteristic of :

a. Manic depressive psychosis with depressive cycle

b. Bipolar-II disorder

c. Catatonic schizophrenia

d. Schizoaffective disorder

Solution. (c) Catatonic schizophrenia


Ref: Read the text below
Sol :
Clinical features of retarded catatonia –
- Mutism
- Rigidity
- Negativism
- posturing (bizarre posture for long period of time
- stupor
- echolalia
- echopraxia
- waxy flexibility
- automatic obedience and verbigeration.

Correct Answer. c

(164). Dapsone is the preferred treatment of all except

a. Pemphigus vulgaris

b. Dermatitis herpetiformis

c. Subcorneal pustular dermatosis

d. Linear IgA disease

Solution. (a) Pemphigus vulgaris


Ref:Read the text below
Sol:
- Pemphigus is best treated with systemic corticosteroids. DH, Linear IgA disease, subcorneal pustular dermatoses respond excellently
with dapsone.

Correct Answer. a

Copyright © 2014 Delhi Academy of Medical Sciences, All Rights Reserved. 75/90
(165). A newborn child born of a non-consanguineous marriage presents with history of recurrent blistering of hands and feet. His father also
gives history of recurrent blisters of hands, feet and trauma prone sites. The diagnosis is

a. Epidemolysis bullosa acquisita

b. Epidermolysis bullosa congenita

c. Porphyria cutanea tarda

d. Bullous lupus erythematosus

Solution. (b) Epidermolysis bullosa congenita


Ref:Read the text below
Sol:
- This child has a genetically inherited bullous disease primarily affecting the trauma prone sites like hands and feet.
- It is therefore a mechanobullous disease suggestive of defective structural proteins from birth, unlike the EB acquisita which is immune
mediated damage to type-7 collagen.
- The condition is calle epidermolysis bullosa congenital which is of further 3 types: simplex
- the most superficial due to defective keratin 5 &14, junctionalis:due to defective proteins like laminin, plectin, integrins etc. and
dystrophic: defective defective collagen 7.
- The level of split in these three are through the basal layer, through the lamina lucida and below the lamina densa

Correct Answer. b

(166). What sign is being demonstrated in Fig 7 ?

a. Nikolsky sign

b. Asboe Hansen sign

c. Auspitz sign

d. Koebner sign

Solution. (b) Asboe Hansen sign


Ref:Read the text below
Sol:
- “Asboe Hansen sign”or “Bulla spread sign”means when you apply gentle pressure on an intact bulla,the fluid spreads away under the
skin away from the site of pressure.
- This spread occurs because of loss of desmosomes(acantholysis).
- This sign is positive in all varieties of pemphigus and many cases of subepidermal blisters, including bullous pemphigoid, dermatitis
herpetiformis, epidermolysisbullosaacquisita, cicatricialpemphigoid, dystrophic epidermolysisbullosa,
- Stevens-Johnson syndrome and toxic epidermal necrolysis.
- Skin biopsy of Pemphigus vulgaris on Direct Immunofluorescence shows “Fish net pattern”

Correct Answer. b

Copyright © 2014 Delhi Academy of Medical Sciences, All Rights Reserved. 76/90
(167). In Pityriasis versicolor infection, the hypopigmented or depigmented macules which are seen are due to :-

a. Degranulation of melanosomes by local immunological action of Malassezia fungus over melanocytes

b. Competitive inhibition of tyrosinase by dicarboxilic acids produced by Malassezia

c. Noncompetitive inhibition of DOPA hydroxylase

d. Competitive inhibition of DOPA hydroxylase

Solution. (b) Competitive inhibition of tyrosinase by dicarboxilic acids produced by Malassezia


Ref: Read the text below
Sol:
- In P. versicolor both hypo & hyperpigmented macles are seen.
- These are mainly distributed over trunk that too over back region most commonly.
- The hypopigmented macules are due to Competitive inhibition of tyrosinase by dicarboxilic acids (Azealic acid) produced by Malassezia
while the hyper pigmented lesions are due to increased size of the melanosomes.

Correct Answer. b

(168). Manak, 18 years old male presents with multiple discrete papules, pustules with erythematous halo over back of shoulders & few
nodulocystic lesions over face with 'ice pick like scarring'. The line of management in this case should be :-

a. Oral azithromycin & topical fuscidin

b. Oral cyanomycin & topical clarithromycin

c. Retinoids

d. Excision & drainage of the nodulocystic leions under high antibiotic coverage with topical clarithromycin on back

Solution. (c) Retinoids


Ref: Roxburgh's -149-157; Rooks p 1953-72
Sol:
- The above case is of acne vulgaris ( nodulocystic variety ) in which now a days the drug of choice is Retinoids ( Isotretinoin ) which is
3rd generation Retinoids & is used orally.
- It is having teratogenic effect, so should not be used in pregnant or hopeful females.

Correct Answer. c

(169). Which of the following statement is correct :-

a. Density of eccrine sweat glands is highest in axilla

b. Body odour is due to eccrine sweat gland secretion

c. Langerhans cells are modified lymphocytes

d. Breast is modified apocrine sweat gland.

Solution. (d) Breast is modified apocrine sweat gland.


Ref: Rooks p 37
Sol:
-Density of eccrine sweat glands is highest in palms & soles
- Body odour is due to Apocrine sweat gland secretion
- Langerhans cells are modified Macrophages

Correct Answer. d

Copyright © 2014 Delhi Academy of Medical Sciences, All Rights Reserved. 77/90
(170). Sweat glands are innervated by :-

a. Sympathetic fibers with Acetylcholine as neurotransmitter

b. Parasympathetic fibers with Acetylcholine as neurotransmitter

c. Sympathetic fibers with Adrenaline as neurotransmitter

d. Both Sympathetic & Parasympathetic fibers with Acetylcholine as neurotransmitter.

Solution. (a) Sympathetic fibers with Acetylcholine as neurotransmitter


Ref: Read the text below
Sol:
- This is very peculiar about sweat glands that though they are having sympathetic innervation but the neurotransmitter is Acetylcholine.
- Also remember that all postganglionic parasympathetic & few postganglionic. Sympathetic (sweat glands & some blood vessels) are
having Muscarinic receptors while ganglia of both sympathetic & parasympathetic are having nicotinic receptors

Correct Answer. a

(171). A 15-year-old boy presents with an as yet unexplained, painless but dramatic swelling of the upper lip. The likely diagnosis is

a. Cellulitis

b. Hereditary angioedema (HAE)

c. Contact dermatitis

d. Angioedema

Solution. (d) Angioedema


Ref: Read the text below
Sol :
- The lip is one of the more common areas involved in angioedema, but many others have been reported, including palms and soles,
where it tends to be slightly tender to touch. Whatever the location, angioedema usually resolves within 48 h, if not much sooner.
- Cellulitis is commonly misdiagnosed, and is incorrect in this case, since it, unlike angioedema, would be red, hot, and tender, and would
probably not resolve within 48 h, even with treatment.
- Hereditary angioedema (HAE) does present with angioedema, but is rare, so is not the likely diagnosis.
- However, possible family history of unexplained respiratory distress needs to be investigated to help rule out HAE.

Correct Answer. d

Copyright © 2014 Delhi Academy of Medical Sciences, All Rights Reserved. 78/90
(172). A patient presents with itchy lesions as shown in Fig 8 . The causes for this morphology is likely to be

a. Sporothrix schenkii

b. Mycobacterium leprae

c. Trichophyton rubrum

d. Candida albicans

Solution. (c) Trichophyton rubrum


Ref: Read the text below
Sol :
- The image is of an annular lesion
- Tinea caused by dermatophytes (Trichophyton/ Microsporum/ Epidermophyton) is one of the commonest causes for this presentation.
- Sporothrix causes linear nodules and sinuses along lymphatics; Candida usually causes white creamy lesions for eg in oral cavity or
erythematous itchy lesions in skin folds/on glans.

Correct Answer. c

(173). Initial treatment of common furuncles of acute onset should include all of the following except

a. Systemic antibiotics

b. Hot compresses

c. Incision and drainage, as indicated

d. Elimination of known etiologic factors

Solution. (a) Systemic antibiotics


Ref: Read the text below
Sol :
- Even though systemic antibiotics are quite commonly prescribed, they are in fact seldom necessary if the other measures, hot
compresses, incision and drainage, and elimination of known etiologic factors, are taken.
- There are exceptions that include furunculosis in immunocompromised patients, in patients with systemic symptomatology, or in severe
cases unresponsive to conservative treatment.

Correct Answer. a

Copyright © 2014 Delhi Academy of Medical Sciences, All Rights Reserved. 79/90
(174). A 30 year old female presentswith well defined,irregular brown red plaques on both shins shown in Fig 9.What could be the possible
underlying systemic disorder she may have?

a. Diabetes

b. Hypertension

c. Hypothyroidism

d. Hyperthyroidism

Solution. (a) Diabetes


Ref:Read the text below
Sol:
- The diagnosis is “Necrobiosislipoidicadiabeticorum”which is seen in young female diabetics.
- Treatment is with steroids,but often unsatisfactory.Chronic stage of this condition may present as ulcers.

Correct Answer. a

(175). A patient of psoriatic erythroderma was being worked up. Pretreatment evaluation revealed an active pulmonary tuberculosis. Most
appropriate medical management would be

a. Methotrexate

b. Cyclosporine

c. Intravenous immunoglobulins

d. Acitretin

Solution. (d) Acitretin


Ref: Read the text below
Sol :
- Methotrexate, cyclosporine and retinoids can be used to control severe psoriasis.
- However, methotrexate and cyclosporine are immunosuppressives and can lead to dissemination of tuberculosis.
-Retinoids are not immunosuppressive and hence will be the first choice in this case. IVIG is not a standard treatment of psoriasis.

Correct Answer. d

Copyright © 2014 Delhi Academy of Medical Sciences, All Rights Reserved. 80/90
(176). Consider the following features :
(1) Paranoid delusions are common.
(2) Early morning walking is characteristically seen.
(3) Memory for recent events is poor.
(4) Reduced libido.
Which of these are the features of depression ?

a. 1 and 2

b. 2 and 3

c. 3 and 4

d. 2 and 4

Solution. (d) 2 and 4


Ref: Read the text below
Sol : Somatic syndrome in depression (ICD-10) : (a) Significant decrease in appetite or weight. (b) Early morning awakening, at least 2
(or more) hours before the usual time of waking up. (c) Diurnal variation, with depression being worst in morning. (d) Pervasive loss of
interest and loss of reactivity to pleasurable stimuli. (e) Psychomotor agitation or retardation.

Correct Answer. d

(177). Astasia-abasia refers to :

a. Choreoathotic gait

b. Vestibular gait

c. Hysterical gait

d. Sensory ataxic gait

Solution. (c) Hysterical gait


Ref: Read the text below
Sol :
- Astasia-abasia – is a typical hysterical gait disorder, usually characterized by wide-based, staggering, jerky, dramatic and irregular gait
with exaggerated body movements.

Correct Answer. c

(178). Panic attack is characterized by all of the following except :

a. Fear of dying

b. Visual hallucinations

c. Choking sensation

d. Fear of losing control

Solution. (d) Visual hallucinations


Ref: Read the text below
Sol :
- Diagnostic criteria for panic attack – A discrete period of intense fear or discomfort, in which four or more of the following symptoms
developed abruptly and reached a peak within 10 min.
1. Palpitations, pounding heart or accelerated heart rate. 2. Sweating. 3 Trembling or shaking. 4. Sensations of shortness of breath of
smothering. 5. Feeling of choking. 6. Chest pain or discomfort. 7. Nausea or abdominal distress. 8. Feeling dizzy, unsteady, lightheaded
or faint. 9. Derealization (feeling of unreality) or depersonalization (being detached from onself) 10. Fear of losing control or going crazy.
11. Fear of dying. 12. Parenthesis (numbness or tingling sensations). 13. Chills or hot flushes.

Correct Answer. d

Copyright © 2014 Delhi Academy of Medical Sciences, All Rights Reserved. 81/90
(179). Drug of choice for panic disorder is :

a. Clonazepam

b. Fluoxetine

c. Chlorpromazine

d. Lithium

Solution. (b) Fluoxetine Ref: Read the text below


Sol :
- Drug of choice for generalized anxiety – benzodiazepines.
- Drug of choice for panic disorder – antidepressants.

Correct Answer. b

(180). ECT is most useful in which type of schizophrenia :

a. Simple

b. Hebephrenic

c. Catatonic

d. Paranoid

Solution. (c) Catatonic


Ref: Read the text below
Sol :

Correct Answer. c

(181). The treatment of choice in the management of acute manic illness is :

a. ECT

b. Amitryptiline

c. Diazepam

d. Lithium

Solution. (d) Lithium


Ref: Read the text below
Sol :
- Drug of choice for acute mania – Li.
- Main stay of treatment in bipolar disorder – lithium carbonate.

Correct Answer. d

Copyright © 2014 Delhi Academy of Medical Sciences, All Rights Reserved. 82/90
(182). The following modes of therapy may be useful for treatment of obsessive compulsive disorder except :

a. Fluoxetine

b. Clomipramine

c. Behaviour therapy

d. Electroconvulsive therapy

Solution. (d) Electroconvulsive therapy


Ref: Read the text below
Sol : Treatment of obsessive compulsive disorders is :
- Serotonine reuptake inhibitor like flluoxetine, fluvoxamine, sertraline,
- Compipramine
- Behavior therapy
- Psychotherapy.
For extreme cases, that are treatment resistant and chronically debilitating, electroconvulsive therapy and psychosurgery are
considerations.

Correct Answer. d

(183). Which one of the following is not correct in cases of bulimia nervosa ?

a. Recurrent bouts of binge eating

b. Lack of self-control over eating during binges

c. Self-induced vomiting or dieting after binge

d. Continuous weight gain

Solution. (d) Continuous weight gain


Ref: Read the text below
Sol :
Bulimia nervosa is an eating disorder characterized by binge eating, or consuming a large amount of food in a short amount of time,
followed by an attempt to rid oneself of the calories consumed, usually by purging (vomiting) and/or by laxative, diuretics or excessive
exercise These cycles often involve rapid and out-of-control eating, which may stop when the bulimic is interrupted by another person or
the stomach hurts from overextension, followed by self-induced vomiting or other forms of purging. This cycle may be repeated several
times a week or, in more serious cases, several times a day, and may directly cause:
- Chronic gastric reflux after eating
- Dehydration and hypokalemia caused by frequent vomiting
- Electrolyte imbalance, which can lead to cardiac arrhythmia, cardiac arrest, and even death
- Esophagitis, or inflammation of the esophagus
- Boerhaave syndrome, a rupture in the esophageal wall due to vomiting
- Oral trauma, in which repetitive insertion of fingers or other objects causes lacerations to the lining of the mouth or throat
- Gastroparesis or delayed emptying
- Constipation
- Infertility
- Enlarged glands in the neck, under the jaw line
- Peptic ulcers
- Calluses or scars on back of hands due to repeated trauma from incisors
- Constant weight fluctuations are common
The frequent contact between teeth and gastric acid, in particular, may cause:
- Severe dental erosion
- Perimolysis, or the erosion of tooth enamel
- Swollen salivary glands

Correct Answer. d

Copyright © 2014 Delhi Academy of Medical Sciences, All Rights Reserved. 83/90
(184). Fear to contamination, counting behaviours, and having to check and recheck are features characteristic of :

a. Panic attacks

b. Agoraphobia

c. Obsessive-compulsive disorder

d. Generalized anxiety disorder

Solution. (c) Obsessive-compulsive disorder


Ref: Read the text below
Sol :
Obsessive-compulsive disorder is an anxiety disorder in which people have unwanted and repeated thoughts, feelings, ideas, sensations
(obsessions), or behaviors that make them feel driven to do something (compulsions). Often the person carries out the behaviors to get
rid of the obsessive thoughts, but this only provides temporary relief. Not performing the obsessive rituals can cause great anxiety.
Causes, incidence, and risk factors Obsessive-compulsive disorder (OCD) is more common than was once thought. Most people who
develop it show symptoms by age 30. There are several theories about the cause of OCD, but none have been confirmed. Some reports
have linked OCD to head injury and infections. Several studies have shown that there are brain abnormalities in patients with OCD, but
more research is needed.
About 20% of people with OCD have tics, which suggests the condition may be related to Tourette syndrome. However, this link is not
clear. Symptoms
- Obsessions or compulsions that are not due to medical illness or drug use
- Obsessions or compulsions that cause major distress or interfere with everyday life
There are many types of obsessions and compulsions. One example is an excessive fear of germs and the compulsion to repeatedly wash
the hands to ward off infection. The person usually recognizes that the behavior is excessive or unreasonable.

Correct Answer. c

(185). Defence mechanism not seen in OCD

a. Isolation

b. Projection

c. Reaction formation

d. Undoing

Solution. (b) Projection


Ref:Read the text below
Sol: Projection
- In Freudian psychology, Psychological projection or projection bias is a psychological defense mechanism where a person unconsciously
denies their own attributes, thoughts, and emotions, which are then ascribed to the outside world, such as to the weather, a tool, or to
other people.
- Thus, it involves imagining or projecting that others have those feelings.It is not a part of OCD.

Correct Answer. b

(186). A Patient with pneumonia for 5 days is admitted to the hospital. He suddenly ceases to recognize the doctor and staff, thinks that he is in
jail and complains of scorpions attacking him. He is in altered sensorium; this condition is

a. Acute delirium

b. Acute dementia

c. Acute schizophrenia

d. Acute paranoia

Solution. (a) Acute delirium


Ref:Read the text below
Sol:
The patient in question has an acute confusional disorder with:
- Disturbance in consciousness (altered sensorium)
- Disturbance in orientation – to place (thinks, he is in jail)
- To person (ceases to recognize the doctor and staff)
- Disturbance in perception (complains of scorpions attacking him)
All these features are consistant with the diagnosis of delirium

Correct Answer. a

Copyright © 2014 Delhi Academy of Medical Sciences, All Rights Reserved. 84/90
(187). A person missing from home, is found wandering purposefully. He is well groomed, and denies of having any amnesia. Most likely
diagnosis is:

a. Dissociative fugue

b. Dissociative amnesia

c. Schizophrenia

d. Dementia

Solution. (a) Dissociative fugue


Ref:Read the text below
Sol:
Dissociative fugue is characterized by
- sudden onset of complete amnesia for his earlier life
o Patient usually wander’s away from home
o Adapts new purposeful identity
o Absence of awareness of amnesia
- All features in the questions are consistend with the diagnosis of dissociative fugue.
- In dissociative amnesia, memory loss is patchy and not complete.
- It is characterized by sudden inability to recall important personal information, particularly concerning stressful or traumatic life
events. Patients does not adapt, any new identity and awareness of amnesia be present in the post amnestic period.

Correct Answer. a

(188). Babu, a 40 years aged male complains of sudden onset palpitations and apprehension. He is sweating for the last 10 minutes and fears of
impending death. Diagnosis is

a. Hysteria

b. Cystic fibrosis

c. Panic attack

d. Generalized anxiety disorder

Solution. (c) Panic attack


Ref:Read the text below
Sol:
A panic attack is a discrete episode of acute anxiety The episode is usually:
- Sudden in onset
- Lasts a few minutes
- Characterized by severe anxiety (indicated

Correct Answer. c

(189). Presently, classification of psychiatric illness is done by :

a. DSM-IV-TR

b. ICD-10

c. Both of the above

d. None of the above

Solution. (c) Both of the above


Ref: Read the text below
Sol :
- The modern era in the classification of mental diseases began towards the end of the 18th century during the French Revolution with
the concepts and humane practices of figures such as Philippe Pinel (1745-1826)
- William Cullen (1710-90) tried to subsume under the heading of ‘neurosis’ all forms of mental disorder including such conditions as
tetanus, epilepsy, vesania, and a rough description of psychosis, but his concepts soon proved overinclusive.
- The term psychosis was first formulated by Ernst von Feuchtersleben (1806-49) in his Principles of Medical Psychology.
- At present, there are two major classifications in Psychiatry, namely ICD-10 (1992) and DSM-IV-TR (2000)

Correct Answer. c

Copyright © 2014 Delhi Academy of Medical Sciences, All Rights Reserved. 85/90
(190). Treatment of reactional neuritis in the acute phase of Type I lepra reaction involves:

a. Active exercises

b. Passive exercises

c. Rest of the affected limb in “neutral” position

d. Surgical exploration

Solution. (c) Rest of the affected limb in “neutral” position


Ref: Read the text below
Sol :
- Lepra reactions especially type I reactions are complicated by neuritis.
- Acute neuritis requires, resting of the affected limb, oral steroids and continued MDT.

Correct Answer. c

(191). Scombroid fish food poisoning caused by proteus spp. mainly producing?

a. Serotonin

b. Bradykinin

c. Catecholamines

d. Histamine.

Solution. (d) Histamine.


Ref: Read the text below
Sol :
- Food-associated flushing may be caused by capsaicin (red pepper), sodium nitrate, or alcohol.
- Alcohol may produce flushing in patients using topical calcineurin inhibitors.
- Sulfites are found in wine, dried fruit, prepared foods, and fresh grapes and potatoes.
- Ciguatera or scombroid fish poisoning is a form of histamine-related food poisoning, caused by histamine within the flesh of the fish

Correct Answer. d

(192). Which of the following is not associated with atopic dermatitis

a. Posterior subcapsular cataract

b. Anterior subcapsular cataract

c. Keratoconus

d. Hertoghe’s sign

Solution. (a) Posterior subcapsular cataract


Ref: Read the text below
Sol :
- Many eye/periorbital changes are associated with atopic dermatitis including:
- Anterior subcapsular cataract
- Keratoconus
- Periorbital pallor (headlight sign)
- Periorbital darkness (allergic shiners)
- Lateral madarosis (Hertoghe’s sign
- Accessory and deep infraorbital lines (Dennies’ lines/ Dennie Morgan folds).

Correct Answer. a

Copyright © 2014 Delhi Academy of Medical Sciences, All Rights Reserved. 86/90
(193). Ramkumar, a 15 year old male is presented to you with complaints of multiple, red and firm papules over his face and mainly around
nose and near mouth. Further examination reveals few lance-ovate hypopigmented macules over trunk. The most appropriate diagnosis
is

a. Acne vulgaris

b. Acne rosacea

c. Perioral dermatitis

d. Tuberous sclerosis

Solution. (d) Tuberous sclerosis


Ref: Read the text below
Sol :
- The combination of facial papules and hypopigmented macules should always lead us to thin of tuberous sclerosis.
- The facial lesions are adenoma sebaceum (angiofibromas) and hypopigmented macules are ash leaf macules.

Correct Answer. d

(194). Which ectoparasitic infestation of skin is shown in Figure 10

a. Sarcoptes scabiei

b. Peiculosis capitis

c. Pediculosis pubis

d. Pediculosis corporis

Solution. (c) Pediculosis pubis


Ref: Read the text below Sol :
- Pediculosis is an infestation of lice—blood-feeding ectoparasitic insects of the order Phthiraptera.
-The condition can occur in almost any species of warm-blooded animal (i.e., mammals and birds), including humans.
- Although "pediculosis" in humans may properly refer to lice infestation of any part of the body, the term is sometimes used loosely to
refer to pediculosis capitis, the infestation of the human head with the specific head louse.
- Head-lice infestation is most frequent on children aged 3–10 and their families. Approximately 3% of school children in the United
States contract head lice
- Females are more frequently infested than males. Those of African descent rarely suffer infestation due to differences in hair texture.
- Head lice are spread through direct head-to-head contact with an infested person. From each egg or "nit" may hatch one nymph that
will grow and develop to the adult louse.
- Lice feed on blood once or more often each day by piercing the skin with their tiny needle-like mouthparts. While feeding they excrete
saliva, which irritates the skin and causes itching.

Correct Answer. c

Copyright © 2014 Delhi Academy of Medical Sciences, All Rights Reserved. 87/90
(195). All the following statements about lichen planus are true except

a. Lichen planus is usually difficult to diagnose.

b. A basic lichen planus lesion is papular.

c. Lichen planus lesions are often distinctively purple.

d. Lichen planus can affect oral mucosa.

Solution. (a) Lichen planus is usually difficult to diagnose.


Ref: Read the text below
Sol :
- Lichen planus is in fact typically easy to diagnose, with its diagnose, with its distinctive purple coloration, popular morphology, and
involvement of the oral mucosa.
- Exceptions do occur; the hypertrophic variety of lichen planus seen on shins can mimic psoriasis.

Correct Answer. a

(196). Which of the following are accepted treatment options for pityriasis rosea ?

a. A group V topical steroid

b. UVB phototherapy

c. No treatment is required, as this is a self-limited disease

d. All of the above

Solution. (d) All of the above


Ref: Read the text below
Sol :
- All of the listed treatment options are acceptable. If the patient is symptomatic, palliative therapy with topical steroids or oral
antihistamines is useful.
- UVB therapy can shorten the duration of the disease process if started within the first week of the eruption.

Correct Answer. d

(197). Tinea versicolor is caused by which of the following organism ?

a. Pityrosporum

b. Microsporum

c. Trichophyton

d. Epidermophyton

Solution. (a) Pityrosporum


Ref: Read the text below
Sol :
- Tinea versicolor is the only tinea infection caused by a non-dermatophyte, pityrosporum.
- The other tinea infections (pedis, corporis, capitis, cruris, unguium) are dermatophyte infections caused by organisms that favor the
keratinized cells of skin, hair, and nails.

Correct Answer. a

Copyright © 2014 Delhi Academy of Medical Sciences, All Rights Reserved. 88/90
(198). A 19-year-old white male presents with complaints of itching and burning of the feet and maceration of the skin between the toes. Which
of the following historical statements would be most helpful to you in developing your differential diagnosis ?

a. History of exposure to Rhus plant species

b. A family member with the same symptoms

c. Daily use of dark-colored socks

d. Participation in school athletics with daily practice sessions

Solution. (d) Participation in school athletics with daily practice sessions Ref: Read the text below
Sol :
- One of the most common sources of tinea pedis infection is the floors of athletic locker rooms and showers. Patients in this age group
frequently get this infection from that source.
- The warm, moist environment promotes tinea growth. Dark socks tend to inhibit resolution of tinea infections, but do not cause them.
- Rhus contact causes a localized reaction of the skin known as rhus dermatitis or poison ivy, oak, and sumac.
- Symptoms include red skin, itching, and vesicle development.

Correct Answer. d

(199). A 2 year old child presents with patches and crusted plaques in face,neck and extensor surfaces of the body.Additional finding is shown
in the Fig 11.What is the diagnosis?

a. Urticaria

b. Seborrheic dermatitis

c. Scabies

d. Atopic dermatitis

Solution. (d) Atopic dermatitis


Ref:Read the text below
Sol:
- The clinical history as well as the finding of Denny-Morgan fold(extrafold of skin beneath the lower eyelid) is suggestive of Atopic
dermatitis.

Correct Answer. d

Copyright © 2014 Delhi Academy of Medical Sciences, All Rights Reserved. 89/90
(200). Post herpetic neuralgia is defined as pain persisting after how many weeks?

a. 1 Week

b. 2 Weeks

c. 3 Weeks

d. 4 Weeks

Solution. (d) 4 Weeks


Ref:Read the text below
Sol:
- Herpes zoster (Shingles) is caused by reactivation of a latent varicella zoster virus in sensory nerve ganglion.
- This reactivation occurs when immunity to varicella zoster virus declines because of aging or immuno-suppression.
- Post herpetic neuralgia is defined as pain persisting beyond 4 weeks since the eruption of acute infective skin lesions.
- Post herpetic neuralgia could last for 3 months or longer.

Correct Answer. d

Test Answer
1.(a) 2.(c) 3.(a) 4.(a) 5.(a) 6.(c) 7.(d) 8.(a) 9.(a) 10.(b)

11.(c) 12.(c) 13.(c) 14.(c) 15.(d) 16.(a) 17.(c) 18.(a) 19.(d) 20.(a)

21.(c) 22.(c) 23.(a) 24.(c) 25.(d) 26.(b) 27.(d) 28.(d) 29.(a) 30.(a)

31.(d) 32.(d) 33.(c) 34.(c) 35.(d) 36.(d) 37.(c) 38.(a) 39.(c) 40.(a)

41.(a) 42.(a) 43.(d) 44.(b) 45.(b) 46.(a) 47.(a) 48.(d) 49.(d) 50.(d)

51.(d) 52.(c) 53.(a) 54.(a) 55.(b) 56.(b) 57.(b) 58.(a) 59.(d) 60.(c)

61.(c) 62.(a) 63.(b) 64.(c) 65.(d) 66.(d) 67.(a) 68.(b) 69.(b) 70.(d)

71.(a) 72.(b) 73.(a) 74.(a) 75.(b) 76.(c) 77.(b) 78.(d) 79.(a) 80.(c)

81.(c) 82.(c) 83.(c) 84.(a) 85.(b) 86.(b) 87.(d) 88.(c) 89.(d) 90.(c)

91.(b) 92.(c) 93.(a) 94.(c) 95.(b) 96.(a) 97.(a) 98.(a) 99.(a) 100.(b)

101.(b) 102.(b) 103.(a) 104.(d) 105.(b) 106.(d) 107.(a) 108.(d) 109.(c) 110.(d)

111.(c) 112.(c) 113.(c) 114.(d) 115.(d) 116.(a) 117.(a) 118.(c) 119.(b) 120.(d)

121.(a) 122.(b) 123.(a) 124.(a) 125.(c) 126.(c) 127.(b) 128.(b) 129.(c) 130.(a)

131.(b) 132.(a) 133.(b) 134.(b) 135.(d) 136.(d) 137.(b) 138.(c) 139.(b) 140.(c)

141.(d) 142.(d) 143.(c) 144.(d) 145.(c) 146.(a) 147.(a) 148.(a) 149.(a) 150.(c)

151.(c) 152.(a) 153.(b) 154.(d) 155.(d) 156.(a) 157.(b) 158.(a) 159.(a) 160.(d)

161.(d) 162.(a) 163.(c) 164.(a) 165.(b) 166.(b) 167.(b) 168.(c) 169.(d) 170.(a)

171.(d) 172.(c) 173.(a) 174.(a) 175.(d) 176.(d) 177.(c) 178.(d) 179.(b) 180.(c)

181.(d) 182.(d) 183.(d) 184.(c) 185.(b) 186.(a) 187.(a) 188.(c) 189.(c) 190.(c)

191.(d) 192.(a) 193.(d) 194.(c) 195.(a) 196.(d) 197.(a) 198.(d) 199.(d) 200.(d)

Copyright © 2014 Delhi Academy of Medical Sciences, All Rights Reserved. 90/90

You might also like